You are on page 1of 440
x30! J2,.3RS REFRIGERATION AND AIR CONDITIONING Second Edition W. F. Stoecker Professor of Mechanical Engineering University of Minow at Urbana-Champaign J. W. Jones Associate Professor of Mechantcal Engineering University of Texas at Austin McGraw-Hill, Inc. New York St. Louis San Fracisco Auckland Bogots (Caracas. Lisbon London Maisid Mexico City Milan ‘Montreal New Delhi San Juan Singapore ‘Sydney Tokyo Toronto wi conTENTS: 240 a 212 243 aaa Das 2416 27 218 219 2.20 221 222 2.23 Chapter 3 34 32 33 34 35 Chapter 4 44 42 43 “4 45 46 47 48 49 Isentropic Compression — Bernoulli's Equation Heat Transfer Conduction Radiation Convection ‘Thermal Resistance Cylindrical Cross Section Heat Exchangers Heat-Transfer Processes Used by the Human Body Metabolism Convection Radiation Evapor Problems References Psychrometry and Wetted-Surface Heat Transfer Importance Psychzometric Chart Saturation Line Relative Humidity Humidity Ratio Enthalpy Specific Volume Combined Heat and Mass Tranafer; the Straight-Line Law [Adiabatic Saturation and Thermodynamic Wet-Bulb Temperature Deviation between Enthalpy and Wet-Bulb Lines Wet-Bulb Thermometer Processes Comment on the Basis of 1 kg of Dry Air ‘Transfer of Sensible and Latent Heat with a Wetted Surface Enthalpy Potential Insights Provided by Enthalpy Potential Problems References Heating- and Cooling-Load Calculations Introduction Health and Comfort Criteria ‘Thermal Comfort Air Quality Estimating Heat Loss and Heat Gain Design Conditions ‘Thermal Transmission Infiltration and Ventilation Loads ‘Summary of Procedure for Estimating Heating Loads 40 40 40 a2 a 43 4a 46 47 48 49 50 St 3 33 54 55 56 58 2 410 at 412 413 414 pada eee ae arararrdes > Chapter 6 6 62 63 4 65 66 or ‘Components of the Cooling Load Internal Loads > Solar Loads through Transparent Surfaces Solar Loads on Opaque Surfaces ‘Summary of Procedures for Estimating Cooling Loads Problems References Air-Conditioning Systems ‘Thermal Distribution Systems Cassie Single-Zone System Outdoor-Air Control Single-Zone-System Design Calculations Multiple-Zone Systems Terminal-Reheat System Dual-Duct or Multizone System Variable-Air-Volume Systems Water Systems Unitary Systems Problems References Fan and Duct Systems Conveying Air Pressure Drop in Straight Ducts Pressure Drop in Rectangular Ducts Pressure Drop in Fittings ‘The ¥2p/2 Term Sudden Enlargement ‘Sudden Contraction ‘Tums Branch Takeofts Branch Entries Design of Duct Systems ‘Velocity Method Equal-Friction Method Optimization of Duct Systems ‘System Balancing (Centrifugal Fans and Their Characteristics Fan Laws Air Distribution in Rooms Circular and Plane Jets Diffusers and Induction Problems References Pumps and Piping, Water and Refrigerant Piping Comparison of Water and Air as Heat-Conveying Media CONTENTS vil 1 1 B ” a 85 86 38 88 39 90 92 95 95 96 7 100 101 101 102 103 103 103 106 109 109 110 i 1B na 16 17 ut 118 19 120 120 123 14 12s 7 127 129 130 130 131 6 REFRIGERATION AND AIR CONDITIONING tional vehicles, tractors, crane cabs, areraft, and ships. The major contstutor 7 tional ing loadin many of these vehics is heat from yolar radiation, and, jn he ure of public transportation, heat from people. The leads are also characterized Sa rapid changes and by a high intensity per unit volume in comparison fo building air conditioning. 116 Food storage and distribution Many meats ish, frat, and vegetable are perish aes eet hei storage life can be extended by refigeraticn. Fruits, many vegeta: apie cexsed meat, such 35 sausages ae stored at temperatres just slighty shove rae ere'te prolong ther life, Other meats, fish vegetables, and fits are fronen 20 eer e cay months at low temperatures until they are defrosted and cooked by the “ire jozen-food chain typically consists of the following links: freezing, strase sn cofngerated warehouses, display ina reigerated case a food markets, and Finally ar regen the home freezer or frozenfood compartment of s domestic rfsges\0r Home tay attempts to feeze food resulted ina product laced with ie crystals Freeing Jos dscovered that the temperature must be plunged rapidly throvah, ihe Hreering vone. Approaches!® to freezing food ineiode atrblast freezing, Where 8 ronunately -30°C is blown with high velocity over Fackages of food stashed Oy arr wallets; contact freecng, where the food is placed between metal plates 8 or Timmersion freezing, where the food is placed n ¢ lowtemperatre brine: sees od icezing, where the individual particles are cased along 3 convey" Dek ae in suspension by an upward-drected stream of sol ir (Fig1-S)iand free dng with eryogenie substance such as nitrogen of carbon dioxide. ening peas on a fidied bed conveyor ball (Lewis Reieaion Compan) Figure 1S APPLICATIONS OF REFRIGERATION AND AIR CONDITIONING 7 Figure 1-6 A refrigerated warehouse. (International Assocation of Refrigerated Warehouses) Storage Fruits and vegetables should be frozen quickly ater harvesting and meats frozen quickly after slaughter to maintain high quality. Truckloed and rallearioad lots are then moved {o refigerated warehouses (Fig. 1-6), where they ae stored at -20 to -23°C, perhaps for many months. To maintain a high quality is fish, the storage tem perature is even lower. Distribution Food moves from the refrigerated warehouses to food markets as needed to replenish the stock there. Inthe market the food is kept refrigerated in display cases heldat 3 t0 °C for dairy products and uifiozen fruits and vegetables at approximately 20°C for frozen foods and ie eam. In the United States about 100,000 refrigerated Aisplay cases ae sold each year. The consumer finally stores the food in a domestic refrigerator or freezer until used, Five milion domestic refrigerators are sold each year in the United States, and for several decades styling and first cost were paramount considerations in the design band manufacture of domestic refrigerators. The need for energy conservation, how: ‘ever, has brought back the engineering challenge in designing these appliances. {8 REFRIGERATION AND AIR CONDITIONING 1:7 Food processing Some foods need operations in ion 9 freezing and refriger- 17 sEeoage ana these processes ental refrigeration 2892) ep produers ‘The chief dairy product are mils 8 Oar ‘and cheese. To pas- Dairy Pek se temperature is elevated to approximately "73°C and held for about 20 tea that process the mik is cooked and ultimately refiigerated to 3 or 4°C for Sorage, In manufacturing ice cream’ the iapesion ot first pasteurized and thor- roy ned. Then, refrigeration equipment cooks He to about 6°C, whereupon aaa aa freezer. The freezer drops the temperature ~5°C, at which temperature i ent ffens but remains fuid enough to flow fro a conlsine- From this point une rece cream i stored below freezing temperatures, he fs evatpundreds of varieties of cheese, cach pera BY © Aifferent.process, tut typical steps include bringing the temperate ‘of milk to about 30°C and then but Seal cubstances, including 2 cheese starter ont sometimes rennet. Part of adding tore alii into the curds, from which the awit ‘whey is drained. A curing evo’ in refigerated rooms follows for most cheeses at temperature of the order of 10°C. 1 aes Refrigeration is esenial in the production of such beverages as concen Beverage ye, beer, and wine, The taste of many drinks cP ‘be improved by serving, them cold nce qoenteates are popular because of thes hich geal and reasonable cost this le expensive to concentrate the jue close 601 orchards and ship it in its fro res ce than to sip the raw fruit, To preserve the Oe ‘of juice, its water must be serena a Low temperature, requiring the entre Poses 1° ‘be carried out at pres: sores much below atmospheric sree brewing industry refigeration controls the ferment reaction and pre: serve, some ofthe intermediate and final products. ANY eet fn the production of seal is Fermentation, an exothermic reaction, FOr producing a lagertype beer Fr sleoion should proceed at 8 temperature between ® fand 12°C, which is maintained ty refdgeration. From this point on in the Prosi he beer is stored in bulk and wlti ~ wy bottled or kegged (Pia, 1-7) in sfrigerated space: pear reason fr cefigerating bakery products 1% provide a better match ‘perween production and demand and thus prevent ‘Many breads and pastries are betwee lowing baking to prowde a longer shel! 1% ‘before being sold to the con: ftonen tq practice that provides freshly baked produc (and the enticing aroma as saranda supermarkets but achieves some Of OF advantages of high produc weld prepare the dough ina central locaton, fez fand then transport it to the Supermarket, where it is baked as needed rath geal and (006 products ae preseted Dy fm ‘rying, in which the produet frozen and then the water i remonee OY ‘ublimation (direct transition from rod water vapor), Te process takes place in a CHU “while heat is carefully appli vee svprodvet to provide the heat of sublimation. Some manufacturers of instar Ceaiee use the freeze-drying proces. 118 Chemical and process industis The chemi] ang PISS industries include th 18 ufacturers of chemicals petroleum refiners, ‘petrochemical plants, paper and pul martes, ete. These industsies require good engineering for their refrigeration sin . [APPLICATIONS OF REFRIGERATION AND AIR CONDITIONING 9 Figure 1-7 Refrigeration is eset in such beverage Indust a Drewes. (Anhewser Busch | Company, Inc) almost evry installation is diferent and the cost of each instalation is so high. Some important functions served by refigeation® in the chemical and process industries | are) separation of gases, (2) condensation of gases, (3) solidification of one sub- Stance in's mixture to separate it from others, (4) maintenance ofa low temperature Statored lguid so that the pressure will ot be excessive, and (S) removal of heat of reaction. "A mixture of hydrocarbon gates can be separated int its sonstituents by cooling the mixture so thatthe substance with the high-temperature boiling point condenses and can be physically removed from the remaining gas. Sometimes in petrochemical plants (Fig. 18) hydrocarbons, such as propane, are used asthe refrigerant. Propane Ir telauvely low in cost compared with other refigerants, andthe plant is completely equipped to handle flammable substances. In other applications separate refrigeration Units, such asthe large packaged unit shown in Fig. 1-9, prove refrigeration forthe proces. 419 Specal applications of reftigeation Other uses of refrgetion and air condition ing span sizes and capacities from small appliances tothe large indostal scale AND AIR CONDITIONING 10 REFRIGERATION ‘ompresior house. cal Corp) a1 29°C, (Refrigeration et tergeation iit for condensing COp Figure 1.9 Twostage packs Enpneering Corporation). [APPLICATIONS OF REFRIGERATION AND AIR CONDITIONING 11 Drinking fountains Small refrigeration units chill drinking water for storage and use ss needed, - Denumdigiers An appliance to dehumidify air in homes and buildings uses a refriger- aavmvnit by fist passing the air to be dchumidifed through the cold evaporator col ae system, whefe the air is both cooled and dehumidified, Then this ool ar lows ver the condenser and is discharged tothe room. se anes ‘The production of ie may take place in domestic refrigerators ice makers wee ng restaurants and motes, and lage industrial fe makers serving food processing and chemical plants sre gurng rinks. Skaters, hockey players, and curlers cannot rely upon the weather to weevide the cold temperatures necessary to freeze the wate in ther fee nis, Pipes rng cold refrigerant or brine are therefore embedded infil of sid or sawdust ver which water fs poured and frozen." aoe vitrton: Refrigeration is sometimes used to freeze sol to fclitate excavations. A Coiresee of refrigeration is in cooling huge masses of concrete! (the chemical rea snwhich occurs during hardening gives off heat, which must be removed so that it vio we cguse expansion and stress the concrete) Concrete may be cooled by chilling sears gravel wate, and cement before mixing, asin Fig, 140, and by embedding chilled-water pips in the concrete saimieny of seawater One of the methods available for desalination of seawater’ is aeine edatwely saltire ice from the seawater, separate the ice, and remelt it to redeem fish water. 140 Precooling materia for concrete in a dam. (Sulzer Brother. Ine) 12. REFRIGERATION AND AIR CONDITIONING 1410 Conelusion The refsgeration and air-condition industry is characterized by Ttay growth. It isa stable industry in Which replacement markets join with new ap- plications to contribute tits health. tons cot of energy since the 1970s hss een a Seni factor in stim- lating technical challenges for te indvidual nest Innovative approaches to wating elficieney which once were considers) impractical now receive serious imprcving co and often prove to be economically jstied. A example is the re cosy of low-temperature heat By elevating the femesrame level of this energy with coreey Sump (which is cefiigeration system). THe Gaye ‘of designing the system of snest st cost with lite or no consideration of the oPaLN cost now seem to be past. REFERENCES 1. “ASHRAE Handbook, Fundamentals Volume Ames £008 of Heating, Reaigeratns, as Conditioning Engineers. Atlanta, Ga 1981 ana ten nope Fundamental” Asdemic New Yo 2 ae tnt iE, Woods: Development of « Hostal Ensy Management Index, MIRAE Trans, vol. 84 pt 2, 1978. ASHRAE Dont priation of Hah emer Iie Sones Hee systers, 3. SiRAb Trans, v0l. 82, pt 1, pP- 31-37. 1976 ASTRA ral nnd Pans, Symp. PHA191, ASHRAE Trans, vol 85. Pt} pp 307-333, 1978. 6: meaohmental Conta fr the Research Laboratory SYP Ag-18-3, ASHRAE Trans. Gaept 1. pp. $11-860, 1978. 1 eRe tabmal Consideration for Laboratory Asim SYP 1B0-15-8, ASHRAE Tron. vol 81, pt. 2, 1975. 0. eke Rooms, ASHRAE Syme. DE-ST-= Cae pdivoning Engncets, Atlanta, Ga 1967, 9 eat Simulation sfdeling of Automobile Com Fw 17, no. 5, pP- 53-85, May 1975 vo oe Prcguct Dect, Applications Volume” P33 society of ‘Handoeuating and A-Condioning Enon Ane 1998, 11 Raed. th Wats Down on, Refligatan, nd AE ‘Conditioners, Symp CH-77-13, RAE Trans, vo. 83, 1s PP. 793-838, 1977 IRAE Tr roa itor, Appcaons Votunes oP: Sg Society of a roe snd nirCondioning Engines, Attn C22 9278 rating ad No mil Mocs ins,” 4th oy McGraw NO American Society of Heating, Reftissating, a8 fort Cooling Requirements, ASHRAE Heating, 13, RN. Shreve an ‘York, 1977. a AETE ana wrotuct Dietor, Apbntions Yann et 55, American Society of “ranabon Tig and AurCondonig Ege Mans Oo 15, Brn Bar Conte Costing on Dam Consrvcton fs WoHls ingest Hydrosectie Power | een Tech. Rev, Yo. 61-80: 1,PP. 319.1979 16, WE, Johnson: Survey of Desain m1 ering Status and Potent, Nel, Water SuP- Ny inpran Aure J rok 40: 2,RP 2-14 Za 197 ; pefore 1981 the actual place of pubeaton for ASS material was New York, batt | queda sare sien for he conennce of readers who HAY Mth wo order from the society. | ~ CHAPTER TWO Ohl. THERMAL PRINCIPLES 21 Roots of refrigeration and air conditioning Since a course in air conditioning and {efvigeration might easily be titled Applications of Thermodynamics and Heat Trans- fer, iti desirable to begin the technical portion of this text with a bref review of the basic elements of these subjects. This chapter extracts some of the fundamental prin- Ciples that are important for calculations used inthe design ané analysis of thermal Satems for buildings and industrial processes. The presentation ofthese principles is ittended to serve a very specific purpose and makes no attempt to cover the full range fof applications of thermodynamics and heat transfer. Readers who feel the need of a tore formal review are directed to basic texts in these subjects.‘~* "This chapter does, however, attempt to present the material in a manner which establishes a pattern of analysis that will be applied repeatedly throughout the re- mainder of the text. This process involves the identification of tie essential elements of the problem or design, the use of simplifications or ideaizations to model the sys tem to be designed or analyzed, and the application of the appropriate physical laws to obtain the necessary result. 2-2 Concepts, models, and laws Thermodynamics and heat transfer have developed from a general set of concepts, based on observations of the physical world, the spe- cific models, and laws necessary to solve problems and design systems. Mass and nergy are two of the basic concepts from which engineering science grows. From our ‘own experience we all have some idea what each of these i but would probably find it difficult to provide a simple, concise, one-paragraph definition of either mass or en- ergy. However, we are well enough acquainted with these concepts to realize that they fare easental elements in our description of the physical world in which we live, ‘As the physical world is extremely complex, itis virtually impossible to describe it precisely. Even if it were, such detailed descriptions would be much too cumber- some for engineering purposes. One of the most significant acccmplishments of engi neering science has been the development of models of physical phenomena which, although they are approximations, provide both a sufficiently accurate description and 8 tractable means of solution, Newton's model of the relationship of force to mass land acceleration is an example. Although it cannot be applied universally, within its ‘ange of application it is accurate and extremely useful 13 14 REFRIGERATION AND AIK CONDITIONING Models in and of themselves, however, ae of lite value unless they can Be © presed in appropriate mathematical tems. The mathemati! expressions of modes Provide the basic equations, or laws, which allow engineering science 40 xPsin Pretict natural phenomena, The fint and second laws of thermodynamice and {he preaitunafer rate equations provide pertinent examples her. In this text we shall be Meing the use of these concepts, models, and laws in the description, desi, and “analysis of thermal systems in buildings and the proces industries * 443 Thermodynamic properties Another essential element in the analysis of thermal 23cm i the identification of the pertinent thermodynamic properties. A ropen) is 2s characteristic or attribute of matter which can be evaluated quantitatively. Ye sataure, pressure, and density are all properties. Workand heat transfer can bs eal aersn eerms of changes in properties, but they are not properties themselves. property i something matter “has.” Work and heat traf are things that a “Con rrorerim to change its properties. Work and heat can be measured only ‘he Koundary of the system, and the amount of energy transfered depends on how = siven change takes place. "a thermodynamics centers on energy, all thermedynamic properties are related to engy, The thermodynamic sate or condition of asystem is defined by the vali ose weoperties. In out considerations we shall examine eqlibium states and Find hat eo imple substance two intensive thermodynamic properties define the ste rae ree teture of substances, e., dry alr and water vapor, its necessary to define thace thermodynamic properties to specify the state, Once the state of the substance rae se Jetermined, all the other thermodynamic properties can be found since they are not all independently variable, "the thermodynamic properties of primary interest in this text are temperature, pressive, density and specific volume, specific heat, enthalpy, enteopy, and the gut ‘vapor property of state Tonperoure ‘The temperature + of a substance indeates its thermal sate and 4 eine to exchange energy with a substance in contact with it, Thus, a substance with a iigher temperature pases energy to one with a lower temperature. Reference point aes Celsos scale ae the freezing point of water (O°C) and the boiling point of water (100°C). " Geolute temperature Tis the number of degress above absolute zero expressed sn kelvine (K); thus T=1°C +273, Since temperature intervals on the two sles are lddential, differences between Celsius temperatures are stated in kelvins srenuie’ pressure p is the normal (perpendicular) force exerted by a fluid per wit area against which the force is exerted. Absolute pressure i the measure of PrESSS sibove sero: gauge pressure is measured above existing atmospheric pressure, “The amit used for pressure is newtons per square meter (N/m), also called @ ‘pascal (Pa). The newton i a unit of force ‘Standard atmospheric pressure is 101,325 Pa = 101.3 kPa. Pramuves are measured by such instruments as pressure gauges Or manometers, shown schematically installed in the air duct of Fig. 2-1. Because one end of the ma aoa open to the atmosphere, the deflection of water in the manometer indicates gauge pressure, jut asthe pressure gauge does. Figure 24 THERMAL PRINCIPLES 15 Manometer Lt Deflection (PY) resi se Air indicating the gauge pressure of air 2 duct with a pressure gauge and a manometer Density and specific volume The density p of a fluid is the mass occupying @ unit volume; the specifi fc volume » is the volume occupied by a unit mass. The density and seeeie volumes are reciprocals of each other. The density of rat standard atmo here pressure and 25°C is approximately 1.2 kgm? Example 2 What is the mass of air contained in a room of dimensions 4 by 6 bby 3 mif the specific volume of the airs 0.83 m?/kg? Solution The volume of the room is 72 m?, and so the mass of arin the room is nr = 86.7 amie Specific hear The specific heat of a substance isthe quantity of eneray requted to sreeihe temperature of a unit mass by 1 K. Since the magnitude of this quantity is influenced by how the process is eared out, how the heat is added or removed must a doceribed, The two most common descriptions are specific Heat at constant volume ¢, and specific heat at constant pressure Cy ‘The second is the more useful to us be Chuse it applies to most of the heating and cooling processes experienced in sir com ditioning a ind refriger ‘The approximate specific heats of several important substances are 4.19 kS/kg-K liquid water 10 KiykgK dry air 1188 KifkgK water vapor where J symbolizes the unit of energy, the joule. Example 2-2 What is the rate of eat input toa water heater if 0.4 ka/s of water ‘enters at 82°C and leaves at 93°C? Solution ‘The pressure of water remains essentially constant as it flows through the heater, 50 is applicable, The amount of energy inthe form of heat added to each kilogram is (4.19 ki/ke-K)(93 ~ 82°C) = 46.1 K/h, 16 REFRIGERATION AND AIR CONDITIONING “The units on opposite sides of equations must balance, Dut the °C and K do can- el becuse the specific heat implies «change in terperctre expres kkelvins and Saga is change in temperature of 11°C. A change of tempers Celsius degrees 3e given magnitude isthe same change in Kelvns, To comes ‘Example 22, con- orn eietpt that O Kgl lows through the heater, The ate of heat input then is (044 kals) (46.1 kif) = 18.44 Kfs = 18:44 KW zs Enthalpy Ff the constant-presure process introduced above is further restricted by permitting no work to be done on the substance, ¢. ‘a compressor, the amount of Ferre or removed per Unit mas ithe change in enthalpy ofthe snes ‘Tables: ae orenthalpy h are avaable for many sbstanecs, Thete ental values are aaa emgnscd on some arbitrary chosen datum plane, For exam, the datum plane weeedtier and steam isan enthalpy value of 2x0 for liqud waved 8 (°C. Based on that 1 wa ese enthalpy of Liquid water at 100°C is 417.06 KI/kg and ‘of water vapor (tear) at 100°C is 2676 kd/ ke ae Ghonge in enthlpy s that amount of est added or remowl Pe unit rasa constant pressure procest, the change i enthalpy of 16 NT! Example sree ig kslkg. The enthalpy property cal also exeress the rates of heat transfer fox proceses where there is vaporization or condensation, € i" 8 ‘water boiler or an airheating coil where steam condenses. Example 2 A flow rate of 0.06 kas of water enters» Bole! 3 90°C, at which eranuaure the enthalpy is 3769 Uke. The water leaves a8 scan at 100°C. What is the rate of heat added by the boiler? “Solution ‘Te change in enthalpy in this constant-presure Process Ah = 2676 - 377 Ki/kg = 2299 KSI ‘The ate of heat transfer to the water in convertingit co steam (0.06 kas) (2299 ki /kg) = 137.9 KW Entropy AXthough entropy + bas important technies! and philosophical connotations Ero Pe oe tia. property ina specific and limited mannes, Pavone does appear in we ay charts and tables of properties and is mentioned hex £0 that it will not be un- rant ae The following are two implications ofthis property: 1. tf a gas or vapor is compressed or expanded fictonesly WNIT adding oF re wr cune eat during the process, the entropy ofthe substance constant. 2 mete proces deseribed in implication 1, the change in ealalPy represents the we enn of work per unit mass required by the compression Of delivered by the expansion. ‘use we shall have for entropy isto read lines of constant Possibly the greatest practical vat compression in vapor-compression 1 entropy on graphs in computing the work frigeration cycles. THERMAL PRINCIPLES 17 Liquid-vapor properties Most heating and cooling systems use substances that pass. between liquid and vapor states in their cycle. Steam and refrigerants are prime ex- amples of these substances Since the pressures, temperatures, and enthalpies are key ‘properties during these changes, the relationships of these propertis ar listed in tables br displayed on charts, e., the pressure-enthalpy diagram for water shown in Fig. 2-2. “The three major regions on the chart are (1) the subcooled-quid region to the Jeft, (2) the liquid-vapor region in the center, and (3) the supetheated-vapor region on the sight. In region 1 only liquid exists, in region 3 only vapor exists, and in repfon 2 both liquid and vapor exist simultaneously. Separating region 2 end region 3 is the ‘aturated-vapor line. As we move to the right along a horizontal line at constant pres- Sure from the saturated-liquid line to the saturated-vapor line, the mixture of liquid fand vapor changes from 100 percent liquid to 100 percent vapor. ‘Thre lines of constant temperature are shown in Fig. 22, ¢= 50°C, t= 100°C, and £7 150°C. Corresponding to our experience, water boils at » higher temperature when the pressure is higher, If the pressure is 12.3 KPa, water bails at $0°C, but at standard atmospheric pressure of 101 kPait boils at 100°C. “Also shown in the superheated vapor region are two lines of constant entropy. 500 00] 200] 200] Sot Liqui'vapot raion ed liquid region 0% 100 80 Presure, KPa 8 50 40 20 Supers heated Taper “TI heen Sarat os OF 10 12 1a 16 18 20 22 24 26 28 30 Enthalpy, Mike Figure 22 Skeleton presurcenthapy diagram for water. 18 REFRIGERATION AND AIR CONDITIONING Example 2-4 If 9 kes of liquid water at SO"C flows into boiler, s heated cared superheated to a temperature of 150°C andthe entre proces takes vince a standard atmospheric presure, what i the rte of heat transfer to the water? Solution “The process consists of thre distin parts: (1) bringing the femperatre sae rabcooled water up to its saturation temperature, (2) converting liquid’ at OA gre inte vapor at 100°C, and (3) supetheating the vapor from 100 to 150°C: sae ce of heat transfer isthe product of the mass rate of flow multiplied by the Thenpe iv enthalpy. The enthalpy of entering water at SOC and 101 KF is 209 ‘Cine wich can be rad approximately from Fig. 27 or detemined more pe: eee ern Appendix Table A-. The enthalpy of superheated steam at 150°C Gnd 101 kPa is 2745 ki/kg. The rate of heat transfer is (9 kg/s) (2745 = 209 kikg) = 22.824 KW Derfect4os law As noted previously, the thermodynamic propertis of substan Nt Peete ependently vaable But ate fixed by the state ofa substance, The iealzed rota at as behavior which ielates the presse, temperatre, and specific volume of a perfect gas provides an example T where p= absolute pressure, Pa ‘y= specifie volume, m°/kg [Re ges constant = 287 I/kgrK for sir and 462 I/keK for water ‘T= absolute temperature, K For our purposes the perfect-gas equation is applicable to iy ar and to highly supe or et ter vapor and. not applicable to water and refrigerant vapors close to thei saturation conditions. Example 2-5 What is the densty of dry air at 101 kPaand 25°C? Solution The density pis the reciprocal ofthe specific volume v, and s0 P 101,000 Pa RT (287 Skg'K) QS +273 K) 18 kgim? 4-4 Thermodynamic processes In discussing thermodynamic properties, we have Fea ay ntrodueed several thermodynamic process (ating and cooling), Pt already ow several more definitions andthe basic models and laws we shal se Before ‘Expanding this discussion to a wider range of applications. mine gy is the central concept in thermodynamics, its fendamental models 394 inns have ben developed to facilitate energy analyses, 10 describe energy conten’ wae nergy transfer. Energy analysis fundamentally an accounting procedure, 20 an ety procedure whatever itis that is under consideration must be clearly ident Lay THERMAL PRINCIPLES 19 fied. In this text we use the term system to designate the object or objects considered in the analysis or discussion. A system may be.as simple as a specified volume of a homogeneous fluid or as complex as the entire thermal-distribution network i a large pulding In most cases we shall define a system in tens of a specified region in space (Gometimes refered to as a contol volume) and enticely enclosed by a closed surface, (Sferred to a3 the system boundary (or control surface). The sizeof the system and the rape of the system boundary are arbitrary and ae specified for cach problem so thet they simplify accounting for the changes in energy storage within the system or energy ronafers across a system boundary. Whatever is not included in the system is called the environment. ‘Consider the simple flow system shown in Fig. 2-3, where mass is transferred rom the environment to the system at point 1 and from the system to the environ- nent at point 2. Such a system could be used to analyze something as simple as 2 Frump of as complex as an entice building, The definition of the system provides the vamework for the models used to describe the real objects considered in thermody: namic analysis ‘The next step in the analysis is to formulate the basic laws sc that they are appl cable to the system defined. The laws of conservation of mass and conservation of nergy provide excelent examples, as we shall be applying them repeatedly in every tapect of air-conditioning and refrigeration design. 2.5 Conservation of mass Mass is a fundamental concept and thus is not simply de- fined. A definition is often presented by reference to Newton's law av Force = ma =m —— @ where m= mass, ke acceleration, ms? V= velocity, m/s = time,s ‘An object subjected to an unbalanced force accelerates ata rate dependent upon the mapnitude of the force. n this context the mass of an object is conceived of as being characteristic of its resistance to change in velocity. Two objects which undergo the same acceleration under action of identical forces have the sare mass. Further, our 6m, im, Figure 23. Consernation of mass in a simple flow system. ——_ 20 REFRIGERATION AND AIR CONDITIONING concept of mast holds thatthe mass of two COIS taken together isthe sum of their Gonespt ones and that cutting a homogen#Ous Body nto two identical parts pro indigo identical asses, each half of the oil mas. ‘Ths idea is the equivalent of the law of conservation of mast. vine present context the principle of comseryaice cof mass states that mass is neither ereated nor destroyed in the processes ‘alyzed. Tt may be stored within 8 SYS; tem or transferred between a system ‘and its environment, but it must be accounted for ter gays procedure, consier Fig, 23 asa TH the system may change ia ato mass flows into oF out of the system. ASUS, that during a time incre- ore yok mass bm, enters to system and a inseTen “bmp leaves. Ifthe mass in Ta ystem a time is mig and that at time 0+ 150 is my » go. conservation of Mass requires that amg +m, = My «50 * M2 Dividing by 60 gives If we express the mass flux a5 om 56 wwe can write the rate of change at any instant a8 im a ifthe rate of change of mass within the system 8 2612, am a 20 and ry =r and we have steady flow. Steady flow will be encountered frequently in our analysis + SteadySlow energy equation In most a-conditong and refrigeration systems eng from one instant to the next (Orit Ney do, the rate pay be assumed to be steady. In the system ipestated as follows: the rate of ® point | pls the eat of energy added a heat mits cof energy leaving at point 2 ‘The mathematical expression energy entering with the stream af ener maf energy pesforming work and minus We Te tual the rate of change of energy i the contol volume. for the energy balance is 4 -(, 8 a apie Demo y+ Pteea)-W en ‘THERMAL PRINCIPLES 21 aw Figure 24 Enerey balance on a contzol volume experiencing steady flow res where *h= mass rate of flow, ke/s athalpy, Ike. V= velocity, mis = elevation, m ravitational acceleration = 9.81 m/s? te of energy transfer in form of heat, W rate of energy transfer in form of work, W E= energy in system, 1 Because we are limiting consideration to steady-flow processes, there is no change of E with respect to time; the dé/d0 term is therefore zero, and :he usual frm of the steady-flow energy equation appears . we B salt, + Bee, Joa (ny Bae) 02) ‘This form of the energy equation will be frequently used inthe following chap- ters. Some applications of Eq. (2-2) will be considered at this point. 27 Heating and cooling In many heating and cooling processes, g., the water heater in Example 2-2 and the boiler in Example 2-3, the changes in certain of the energy terms are negligible. Often the magnitude of change in the kinetic-energy term V2/2 and the potential-energy term 9.812 from one point to another is negligible compared with the magnitude of change of enthalpy, the work done, or heat transferred. If no work is done by a pump, compressor, of engine in the process, W=0. The energy equation then reduces to tinh, = tity or = 7h, ~hy) 4, the rate of heat transfer equals the mass rate of flow multiplied by the change in enthalpy, as assumed in Examples 2-2 and 23 Example 2-6 Water flowing at a steady rate of 1.2 kgls is to be chilled from 10 to 4°C to supply a cooling coil in an air-conditioning system. Determine the neces- sary rate of heat transfer. ——_—_— 22. REFRIGERATION AND AIR CONDITIONING ‘solution From Table A-1, at °C h= 1680 ka/ke and at 10°C A= 41.99 kof. ‘Then - qth, hy) = (12k (1680 ~ 41.9) - 30.23 KW 8 Adiabatic processes Adiabatic means that no heat By transferred; thus @= ae that are esentally adiabatic occur when the wails 9f system are ther very aiated. Even when the walls are not insulated Oe throughput rates of ay ine large in elation tothe energy transmitted to from the environment in ere Pn of heat, the process may be considered adiabatic, 2.9 Compression work An example of process which => vapor 120°C, 250 kr — . Cc IN 101.3 kPa Presute 013 ir reducing valve LAtaaanannrnsen} = seit” Figure 214 Pressuezeducing valve in Problem () What is the state of the water entering the vale (subcooled liquid, saturated Liquid, oF vapor)? XG) For each kilogram that enters the pressure-educing valve, how much leaves the separating tank as vapor? Ans, 0.0375 2.2 Air flowing at a rate of 2.5 kes is heated in a heat exchanger from -10 t0 30°C: ‘What is the rate of heat transfer? Ans. 100 kW 23 One instrument for measuring the rate of airflow is a venturi, as shown in Fig, isc where the cross-sectional area is reduced and the pressure difference between postions 4 and B measured, The flow rate of air having a density of 1.15 kelm? isto Permeasused in a venturi where the area at position A is 0.5 m? and the area at B is bee 2 The deflection of water (density = 1000 kg/m?) in a manometer is 20 mm The Tiow between A and B can be considered to be frictionless so that Bernoulli's equation apples. (a) What isthe pressure difference between positions A and B? (b) What is the airflow rate? Ans. 12.32 m/s 24 Use the perlect¢as equation with R = 462 I7ke* K to compute the specific ZShume of saturated vapor water at 20°C. Compare with data of Table A:I. Ans Deviation = 0.19% 28 Using the relationship shown on Fig. 26 for heat transfer when a fluid flows in = aide a tube, what is the percentage increase or decrease in the convection heat-transfer oetticient hy ifthe viscosity of the Muid is decreased 10 percent? Anz, 4.3% inerease ‘2s What is the order of magnitude of heat release by convection from a human body sohen the air velocity is 0.25 m/s and its temperature is 24°C? Ans, 60 W 3:7 What is the order of magnitude of radiant heat transfer from a human body in ¢ ‘Comfort air-conditioning situation? Ans. 40 W ae Aistgw + te 20mm ee a 4 pa iiskaim? —_* Figure 215-A ventrl for measuring ac fow, ‘THERMAL PRINCIPLES 39 28 What isthe approximate rate of heat loss due to insensble evaporation ifthe skin temperature is 32°C, the vapor pressure is 4750 Pa, and the vapor pressure of air is 1700 Pa? The latent heat of water is 243 MI/Kg; Cayy = 1.2 X 10™? g[Pa +s» m? Ans. 18W REFERENCES 1. G. J. Van Wylen and R. E. Sonntag: “Fundamentals of Classical Thermodynamics," Wiley New York, 1978 2. W. D. Reynolds and H.C, Perkins: “Engincering Thermodynamics" McGraw-Hl, New York, 1970. K. Wark: “Thermodynamics,” 2d ed, MeGrawtill, New York, 1976 J.P Holman: "Heat Transfer,” 4th e, MeGra0 Hil, New York, 1976 F.Kreith and W, Z, Black: “Basic Heat Transfer Hayper & Row, Nex Vork, 1980. [ASHRAE Handbook, Fundamentals Volume,” chap. 8, American Society of Heating, Refi erating, and Air Conditioning Enginers, Adana, Ga, 1981 (CHAPTER " THREE ee PSYCHROMETRY AND WETTED-SURFACE HEAT TRANSFER 44.4 Importance Paychrometty is the study of the BOPeTS, of mixtures of air and donee, The subject is important in airconditioning Pacts because atmospheric water waPor pletely dry but a mixture of arand watt vapor. oh airconditioning proceses water is removed from the a-waieh Pet mixture, and ia others water i rineal Payehrometric principles are applied in ates Sapir this book, eg. to load saat og, airconditioning systems, coolingand debumiciNing ‘oils, cooling towers, and evaporative condensers ramon equipment there is a heat and masetransier Pst ‘between air and a wetted sorface, Example include some types of hunidiier, ‘dehumiaifying and cool- eg us and waterspray equipment such as cooin O¥% and evaporative condens ine ie convenient reatons can be developed toexpees ee fof heat and mass ity potent, discussed Iter in his chapter, But fist psychro tuansfer using enthal by property, followed by a discussion of the most metric chart is explored, property Common air-conditioning processes. ‘32 Paychrometric chart Since charts showing perchromeltt properties are readily 32 Peteag 31), why should we concern ourselves with te ‘development of a chart’ sae me fe to become aware of the base ofthe chat a ‘be able to calculat a ats of conditions, eg, nonstandard barometic Pes is epbyatep development of the psychrometsc hat that follows will mak ase of a few simplifying assumptions. They wil be point or ‘long the way wit a mote accurate caesatin, The chart that can Be deve properties at recommendations 42 REFRIGERATION AND AIR CONDITIONING 4 ‘Watervapor presse, KPH | ‘Superheated vapor “Temperture, °C Figure 32 Sevration line oped using simple equations is reasonably accurate and an BE used in most engineer sep caleulations, but of course the most accurate chart or date ‘auilable should be used. We srration line The coordinates chosen forthe payehrometrc Shae presented in ae Shapter are the temperature ¢ for the abscss ané tempor the water-vapor pressure p, fr the ovdnat. Fist consider the chat to PIAS water alone. The sat- aa ge can now be drawn onthe chart (Fg. 3:2) Data for the startet line ean bration es deely rom tables of saturated water (Table 4-1) The 10 the vier of the saturation line represents superheated water vapos f superheated vapor amsoled at constant pressure, it wil eventually teach the saturation line, where it begins to condense. a coy sic has been present with the water vapor. What the ef Fig, 4.2 ue ig present? Ideally, none, The water vapor coatinus o Beh ‘though no i; weve present. At a given water-vapor pressure, which fs n0W A partial pressure, cor air were Pes atthe sre temperature as it woul i no at were Prag ‘There erty ba alight interaction between the molecules of 2s and Wt M001, which panes the steam-table data slighty. Table A-2 presents he ‘properties of air alt= nang sh water vapor. A comparison of vapor pressures of the Yous! the air mix: cated wpe, A with that of pure water shown in Table Ac eves practically no idifference in pressure at a given temperature ‘Tipure 3.2 ean now be considered applicable to wm ar-water vanes sixture. The portion of the chart now of significance is bounded by the saturation line and the orto the condition of the mixture les onthe saturation ling 0° 2 stid to be see eds meaning that any decrease in temperature will resi condensation of Seated yapor into liquid. To the ight of the saturation line the at fs unsaturated, Tripoint A represents the condition of the ai, the temperate of that mixture Wl ie ts be reduced to temperature Bin order for condensation to Bi. Air at “Fis sad to have a dew-point temperature of B. “44 Relative humidity The slative humidity ¢ is defined as the TNS of the mole Teotion of water vapor in moist air tomole faction of water vapor saturated alr at yo2909 22939990 PSYCHROMETRY AND WETTEDSURFACE MEAT TRANSFER 43, Saturation tine Watervapor pressure, KPa Tetatve humidity 6 = 05 Temperature, Figure 33. Relativehumiity tine. the same temperature and pressure, From perfectgas relationships another expression for dis existing partial pressure of water vapor Jaturation pressure of pure water at same temperature Lines of constant relative humidity can be added to the chart asin Fig. 3-3; by mark: ing off vertical distances between the saturation fine and the base of the chart. The felative humidity of 0.50, for example, has an ordinate equal to one-half that of the saturation ine at that temperature 35 Humidity ratio The humidity ratio W is the mass of water interspersed in each Jblogram of dry alt. The humidity ratio, lke the next several properties to be studied enthalpy and specific volume-is based on 1 kg of dry at. The perfect-gas equation can be summoned to solve for the humidity ratio. Both water vapor and air may be Goumed to be perfect gases (obey the equation py= RT and have constant specific heats) in the usual air-conditioning applications. Airis assumed to be a perfect gas because ils temperature is high relative to its saturation temperature, and water vapor fe asumed to be a perfect gas because its pressure is low reative to its saturation pressure kgof water vapor _,VIR,T __ PylR, [porary air, VIR,T (,-P)IR, a where W= humidity ratio, (kg of water vapor)/(kg of dry ar) V-= arbitrary volume of air-vapor mixture, m? y= atmospheric pressure =p, + Py, Pa partial pressure of dry ar, Pa gas constant of dry air= 287 3/kg*K gas constant of water vapor = 461.5 J/kg*K absolute temperature of ai-vapor mixture, K 44 REFRIGERATION AND AIR CONDITIONING ‘Hamidi ration, KaPe8 fo Figure 34 Humidity ratio W as 2 ‘other ovat "Temperature, Substituting the numerical values f Ry and R, ito Ea. (3-1) Bes oat yo 2 Pe wasn Pe on has now appeared on the soent, and from this point on rhrometric chart the chart willbe unigue f0 8 Be? in sine pressure. Equation (32) shows the relationship be'wer the humidity ratio ara re terapor pressure, so that companion seals can be DOW” ‘ordinates of ihe poyehtometic chart, a8 ilustrated in Fig. 3-448 "a (G2) shows, the relation be: ire Pi and p, isnot perfectly Hear. n Fig. 3-1 and tn ost psychrometsic charts eee eg Sued linearly, which makes the Psa sighly ontnes ‘The atmospheric pressure Pr jn the development of the Ps) 60 percent relative humidity Example 3-1 Compute the humidity ratio of sir at ssure is the standard value of chan the temperature is 30°C. The barometric, res 101.3 kPa. at 30°C is 4.241 kPa from ressure of saturated air ‘the water-vapor pressure of Solution The water-vapor le A-L. Since the relative humidity is 60 percent, rao i 0.60 (4.241 KPa) = 2.545 kPa. From Eq,(3-2) 2545 = 00160 kek 545, oe ‘This esult checks the value read from Fig. 31. 446 Enthalyy The enthalpy ofthe mixture of dry ar ard wales the sum of a Mthalpy of the dry air and the enthalpy of the watet per” Enthalpy values are {hee pased on some datum plane, and the zero value of dry air is chosen a8 air alvaye bigs sur walu of the water vapor is satucated bguid wet (°C, the same oe pane that i used for tables of steam. An equation fr the ‘enthalpy is d290d 39999392 ‘ PSYCHROMETRY AND WETTED SURFACEMEAT TRANSFER 45 Saturation line of superheated vapor Tine of constant temperature Enthalpy, Kk Enwopy, Kika K Figure 35 Line of constant temperature shows that the enthalpy of superheated water vapor i approximately equal to the enthalpy of saturated vapor at the sme tempers, ft Wh, klk dry air G3) were ¢, © specific heat of dry alt at constant pressure = 1.0 kS/kg-K, temperature of airvapor mixture, °C fn, = enthalpy of saturated steam at temperature of ai-vaper mixture, kJ/kg Equation (33) gives quite accurate results, although several refinements can be made. ‘The specific heat c, actually varies from 1.006 at 0°C to 1,009 at SO°C. The enthalpy cof water vapor fy fs for saturated steam, but the water vapor in he aivapor mixture js likely to be superheated. No appreciable error results, however, because of the for- tunate selationship of enthalpy and temperature shown on the Mollie diagram of Fig 35, A line of constant enthalpy can now be added to the psychrometsc chart, as in Fig. 36. Suppose, for example, that the 95 kJ/kg enthalpy line is to be constructed ‘Several arbitrary temperatures. can be chosen and the humidity ratio computed at 95 k/kg using Eq. (3-3). The humidity ratio thus computed and the temperature Jocate one point on the line of constant enthalpy. Homiaity 0, Keke Tine of constant enthalpy Temperature, °C igure 34 Line of constant enthalpy. 46. REFRIGERATION AND AIR CONDITIONING ‘Example 32 Locate the point on the 95 kkgenthalry ine where the tempera ture is 50°C. ‘Solution At 1= 50°C, hy (8.3) for h= 95 KS/kg gives 592 ks/kg from Table A-!. Solving for W from Ea 95 - 1.0(50) 951.009) _ 90 p Foy 700174 Kelle ‘The lines within the confines of the saturation line and the axes on Fig. 3-1 are rnot the lines of constant enthalpy but lines of constant wet-bulb temperature (See. 3.9), which deviate slightly from lines of constant enthalpy. Lines of constant en ‘halpy are shown to the left of the saturation line in Fig, 3.1, and ther continuations arevown at the right and bottom borders of the chart. The procedure for reading tnthalpy values off the psychrometric chart wil be explained later. 3.7 Specific volume The perfect gas equation is used to calculate the specific volume atthe airvapor mixture. The specific volume isthe number af cubic meters of mixture fet kilogram of dry ir. It could just 8 well be the cuble mes of ry air o the bi peters of mixture per kilogram of dry air, since the volumes occupied by the individ tual substances are the same, ‘From the perfect gas equation, the specific volume vis RIT_RT 3 yal AeX ong dry at G4) Py Pro Ps ‘To establish points on. line of constant specific volume, 0.90 m3/kg for example, sub- Jyats 0.9 for », the barometric pressure for p,, and at arbitrary values of T solve for dy The pairs ofp and values then deserbe the line of constant asin Fig. 37. Example 33 What isthe specific volume of an at-water-vapor mixture having 3 temperature of 24°C and a relative humidity of 20 percent at standard barometric pressure? Tomy rato, Ka Figue 3-7 Line of constant specifi "Temperature," volume, ae od = — 9 11 tempenties, . ee ‘ater TE td sta Solution The water-vapor pressure of saturated air at 24°C is, from Table A-1, 2.982 kPa; so the vapor pressure with a relative humidity of 20 percent is 10.2(2.982) = 0.5964 kPa = 596.4 Pa. Applying Eq. (3-4), we get 287(24 + 273.15) 101,300 596 ‘This result checks the value from Fig. 3+ = 0.85 m?/kg dry at 3.8 Combined heat and mass transfer; the straightdine law The final psychrometric property to be considered is the wet-bulb temperature, but in order to improve our nderstanding of this property a short detour will be made. It leads into the combined process of heat and mass transfer and proposes the straight-line law. This law states that when air is transferring heat and mass (water) to or from a wetted surface, the Condition of the air shown on psychometric chart drives toward the saturation line at the temperature of the wetted surface. If air flows over a weited surface as in Fig. 13:8, the condition of air passing over differential area dA changes from condition 1 to ‘Condition 2 on the psychrometric chart, Fig. 3-9. The straightline law asserts that point 2 lies on a straight line drawn between point 1 and the situration curve at the ‘wetted surface temperature ‘tis no surprise that the warm air at 1 drops in temperature when in contact with ‘water at temperature f,,. It is also to be expected that the ai at 1, having @ higher Figure 39 Condition of air drives toward saturation line at temperature of wetted Temperate, "© surface 48 REFRIGERATION AND AIR CONDITIONING i, sony. n ik ipa, q i ne@) Temas tas woh meets nem NY Sythe igure 340 Adiabatic saturation. vapor pressure than the liquid at temperatare fw will transfer mass by condensing are eager vapor and dzopping the humidity rato of Me ‘ir. What is unique is that some wate peat and mast ansfer are so elated thatthe Fath a straight line diving ‘be ted the saturation line atthe wetted surface expert ‘This special property is Reto the value of unity of the Lewis relation, a ‘dimensionless group that will be © plained in Sec. 3-14. +39 Adiabatic saturation and thermodynamic wet bulb lenperir, ‘An adiabatic sat 38 Aa(tag. 510) isa device in which ai lows throygh 0% fof water. The water wifi eontinvousy, andthe spray provides #0 UC ‘surface area that the ait leaves Srey chamber in equibsurm with the water, WN espect to both temperature toe par pressure. The device i adiabatic in that He walls of the saturator are inst ane apt no heat is added to, or extracted from, We WO Tine that circulates the Ite om the sump back tothe sprays In order 0 Pern the process itis neces: wie provide makeup water to compensate fo 6 Fmount of water evaporated int sy Foe temperature of this makeup water is contelled that it is the same 25 ‘that in the sump. aeros the adiabatic saturator has achieved a steady 05 ‘condition, the temper ure indicated by an accurate thermometer immeney it the sump is the thermody’ cr ver bulb temperature. Certain combinations oF 2 conditions wall result im & ‘jren sump temperature and can be defined by “writing an energy balance about the en aris energy balance, waten onthe basi of Wt mash flow of air, is ay they (y= MD @s) winery's the enbaly of saturate iui a the Sun oF thermodynamic wee-bulb temperature. PSYCHROMETRY AND WETTEDSURFACE HEAT TRANSFER 49 Line of constant enthalpy Line of constant wet-bulb temperature Figure 311 Line of constant ‘thermodynamic wet-bulb tem “Temperature, °C perature Wer-bu temperatere— | (On the paychrometric chart in Fig. 3-11, point 1 lies below the line of constant ‘enthalpy that pastes through point 2. Any other condition of ai that results in the ‘same sump temperature, such as point 1’, has the same wet-bulb temperature. This line is straight because of the straight-line law, which states that the entering air at point 1 drives toward the saturation line at the wettedsurface temperature. The straight ine between points 1 and 2 represents the path of the ai as it passes through the saturator. ‘Lines of constant wet-bulb temperature are shown on psychrometric charts, as in Fig. 3-1, but lines of constant enthalpy are rarely shown. The enthalpy scale tothe left of the saturation line apples to air that is saturated. For unsaturated air the en- thalpy scale on the left must be combined with the enthalpy scale shown at the right and bottom borders of the chart. The deviation between the enthalpy and wet-bulb- temperature lines will be explained next. 3-10 Deviation between enthalpy and wet-bulb lines As Fig 311 indicates, readings of enthalpy obtained by following the wet-bulb line to the saturation curve specify ‘values of enthalpy that are 100 high. The psychrometric chart, Fig 3-1, shows lines bf constant thermodynamic wet-bulb temperature and not ins of constant enthalpy. ‘The enthalpy scale shown at the left applies only to the conditons on the saturation line, and both the scale at the left and the scales at the right and bottom borders shouldbe used for more precise determinations of enthalpy. ‘To check an enthalpy deviation, compare the chart reading witha calculation for sir having a dry bulb temperature of 40°C and a relative humidity of 41 percent. The wet-bulb temperature of air at this condition is 28°C. In Fig. 31 straightedge can beset at 40°C dry-bulb temperature and 41 percent relative humidity and pivoted about that point until the enthalpy values on the left and right enthalpy sales match. That value is 89 K/kg. Equation (3-5) permits calculation of the enthalpy of the point in question, fy, by comecting the value of h (the enthalpy of saturated air at the same wet-bulb temperature) ‘hy = 89.7 kk (Wy ~ M4) 50 REFRIGERATION AND AIR CONDITIONING Wes ® o gue 3:12 (0) The wetulbempertre, and (0) the procs on Paehromsi al were Wy = 0.019 klk Wh = 0.0241 kelks hiy= gat 28°C = 117.3 Kk nf = $0.7 ~ 117 3(0.0241 - 0018) 9.1 kik, 4411 Wet-ulb thermometer Although the adiabatic saturator of Fit 3410 is not a a omient device for routine measurement, a thermometer NeVinE wetted wick, as cane, would be convenient, We must therefore desermine Whether the wetbulb in Fie aly indicates the thermodynamic wetbulb temperate: te wetted {herr e wik I finite, rather than infinite Like the saturator fo Te 3-10, s0 the strange in state of air passing over the wetted bulb can 2° represented by process 1-2 Seg. 3-12b. Since the energy balance about the bulb is iy Wyle hy * Way points | and 2 ie on the same thermodynamic wet bulb line, The important ques- potnts Mvcver is: What isthe temperature of the water on {he ‘wick? The answer, tion, ower rom the application of the straight law is that the conten of the eng at point 1 has been driving toward the saturation line 2 temperature ai ara rarface in order to reach pint 2. Had rove wet’ ‘surface been avail: Shi 'ue state of the air would contin to dive along the straight line toward the saturation curve. ei his pioneer paper on paychrometry, assumed that 2 temperature of water on a wet-ulb thermometer was the same ax that in 9 ‘adiabatic saturator. ot ae ofo2 grouped the terms that bear his name and conelvde that a value of aly of this dimensionless group results in identical temperate cof a wetted wick wr Giabatic spray. In 1933 Lewis? demonstrated that it ‘atmospheres other than a aiater vapor the reading of a wet bulb thermometer nd ‘he saturated spray si aifroven, We shall hereafter consider the tempersture of he wet-bulb thermom- ate dierene adiabatic spray to be the same and drop the quaificaion “thermody: a the wet bul temperature, simply eaing it she we-bl temperature. DEL pr29009 PSYCHROMETRY AND WETTED SURFACE HEAT TRANSFER SI ami tale Figure $13 Sensible heating or cooling Figure 3:14 Humiliation 3.12 Processes Processes performed on air can be plotted on the psychrometric chart for quick visualization, OF even more importance isthe fact thatthe chart can be used to determine changes in such significant properties as temperature, humidity ratio, and enthalpy for the processes. Some of the basic processes will now be shown, including i) sensible heating or cooling, (2) humidification, adiabatic and nonadiabatic, (3) cooling and dehumidification, (4) chemical dehumidification, and (5) mixing. 1. Sensible heating or cooling refers to a rate of heat transfer attributable only to a change in dry-bulb temperature of the air. Figure 313 shows a change in dry-bulb temperature with no change in humidity ratio. 2. Humidification, as shown in Fig. 3-14, may be adiabatic a shown in process 1-2, or with addition of heat, asin process 1-3. 43. Cooling and dehumidification results ina reduction of both the dry-bulb tempera: ture and the humidity ratio (Fig. 3-15). A cooling and dshumidifying coll pet~ forms such a process. The refrigeration capacity in kilowatss during cooling and ‘dehumidifying process is given by Refsigeration capacity = w(hy ~ty) ‘where w isin kilograms per second and fy, and ty in klojoules per kilogram. Figure 3.15 Cooling and dehumidifcation 52 REFRIGERATION AND AIR CONDITIONING 3999909 aN sishumidiea nidiy ati, ks 39933399 “Temperatures Figure 3:16 Chemical dehumidification 44, In the process of chemical dehumidifiction (Fig. 346) the water vapor from the iris absorbed or adsorbed by a hygroscopic material, Since the process, if ther, nally iolated, is essentially one of constant enthalpy, and since the humidity Tato decreases, the temperature of the air must increase. 5. Mixing of two streams of air is a common process in sir conditioning, Figure 3.17 Shows the mixing of w, kg[s of air at condition 1 with w kg/s of air at condition 2 The result is condition 3, shown on the psychroretsic chart in Fig. 3-17. The fundarnental equations applicable to the mixing process are an energy balance and ‘a mass balance. The energy balance is : why # Wolly = OF, # Was G6) ‘and the balance of the mass of water is 1, My wy My = (or, + WM en "Temperature," © © agement of mixing process. () Mixing proces on psychrometsc } Figare 3.17 (@) Schematic 2 chart < ‘ PSYCHROMETRY AND WETTED SURFACEHEAT TRANSFER 53 Equations (3-6) and (3-7) show that the final enthalpy and humidity ratio are weighted averages of the entering enthalpies and humidity ratios. An approxima- ton used by many engineers is that the final temperature and humidity ratio are weighted averages of the entering values. With that approximation, the point vn the psychromettic chart representing the result of # mixing proces lis on Straight line connecting the points representing the entering conditions. Further, rare, the ratio of distances on the line, (1-3)/(2-3), equals the rato of the flow rates, wa/Wy. The error of this approximation is caused by the variation in spe- Cifc heats of moist air and is usualy les than 1 percent. 3.13 Comment on the basis of 1 kg of dry air The enthalpy, humidity ratio, and spe- ‘ie volume are all based on 1 kg of dry air, and it may seem strange to speak of, say, he mass of water in 1 kg of dry al. (Correctly the humidity rato should be expressed ir the mass of water associared with I kg of dry ar.) A review of some of the processes presented in Sec. 3-12, however, shows the usefulness ofthe tasis of dry ar. Inthe Processes shown in Figs. 3-14 to 3-16 the total mass changes throughout the process aretusr of the addition or extraction of water. If total mass of mixture were used as te basis, it would be necessary to recalculate the mass flow ate after each of these processes, The flow rate of dry air, however, remains constant trough the processes 3.14 Transfer of sensible and latent heat with a wetted surface When air flows past etted surface, 23 showin in Fig. 3-18, there is a likelihood of transfer of both sensible wei atent heat. If there is a difference in temperature between the air fg and the ttted surface f, heat wil be transfered. If there is difference in the partial pres Wie of water vapor in the ar p, and that of the water 7, there wil be a transfer Sf mass (water) This transfer of mass causes a thermal-energy transfer a well, ecause Sf vapor condenses from the aig the latent heat must be removed at the water. Con- ‘Tesely, if some liquid evaporates from the water layer, the latent heat ofthis vapor- ined water must be supplied to the water. The rate of sensible-heat transfer from the water surface to the alr q, can be cal- culated by the convection equation aq, 2h, 4A (Gt) Gs) where qy= rate ofsenible-heat transfer, W ‘hg = convection coefficient, Wim?+K ‘A= area, m? ae Figure 3-18 Heat and mass transfer between at lan! nda wetted susface —— $4 REFRIGERATION AND AIR CONDITIONING \ rere onan ann he wert operon oe scene tence, yy ag econ 35 Howe hat he ely ees i i vapor psu, and sa coneponding popetonal <2 be established rate of mass tansfor= hip 4A(W,-,) els “where hp = proportionality constant, kg/m? Mp Riya of murda tweed suis tempera since eed too fom the wae anes» wane of eat due 19 he Sondensation or evaporation, a, = hy 44, WH 6s) te of latenttheat transfer, W I= att heat of water a PKB cif naugh may sem to determin acute aus of the cone Af gra particular situation, many more dat =r oe ete he fot & ble ent teaser tha fr the masstransfe roressonst costes nartely the ansport mechani athe wel ES controls the stant replat afr ie same one that cons the ae 9 nat transfer. ae on indeed des, ost proportional ean between hp oA ‘Some These shes ofthe boundary layer analy are ivenn Ref, 4. This proportionality is expressed by where ap G10) i fom wher Gp the specific heat of mess ai, KE Tp cheat of molt alts based on 1 Kg of dry ais ands thus the sum of the specific heat of dry air and that of the water vapor 1 com = op* Moe eu) i 4.48 Entiapy potent The concept of entaipy pont is weal ons ST i 2. he ruse of total est eae ps nt in os Pore ‘nnd compo: \ yng the ay ect contact bebwen air and water, The xDreton transfer qi ae ae ta ou a ferential aan dis avalible fom a combiason of Eas. G8) and 9) dag = dq, + da hg AA 4) HA MMe “Applying the expression for hp from Ea. 3-10) ves h \ cA td MO Whip PSYCHROMETRY AND WETTEDSURFACE HEAT TRANSFER 5S Substituting Eq, (3-11), we get Weepgti* Wyhyg) G12) haa 4, = egt+ Wg) Cytet Mbp om ‘The final approximation is to add to Bg. (3-12) an expression that is almost nepigible compared with the other terms. That expression is W/ty- Wah, where hy” B'ihe enthalpy of ssturated Ligud water at temperature f, Equation (12) then becomes Aaa legis + My + gl ~ Leyla + Maly +g 6, p40) 13) ‘the expresion in the ft se of brackets of a, (3-19 prey the enthalpy of are creat th wetted ifn temperature end the expeston in the second set St pctets i precy the enthalpy ofthe ar inthe ie ean, The unt of both haha are ous per klograr of ry ar Thus a A 4-4) Gs) Spm ‘The name enthalpy potential originates from Eq. (3-14) tecaise the potential for the transfer of the sum of sensible and latent heats i the dierence between the enthalpy of saturated air at the wetted surface temperature h, and the enthalpy of ar inthe fee steam h, "The specific heat of molt air py is expresed by Eq. (311), but for states of, sir near those of normal room conditions a value of 1.02 Ki K may be used. For example, with air at 25°C and 50 percent relative humidity, ¢, = 1.00 ki/ke-K, 10.011 Kehkty py = 188, and Gym = 1.0207 KS /kg“K. 3.16 Insights provided by enthalpy potential In addition to helping quantify the calculations of heat and mass transfer in cooling and dehumicifying coils, sprayed coils, evaporative condensers, and cooling towers, the enthalpy potential provides a ‘qualitative indication of the direction of total heat flow. Thee different cases are illustrated in Figs. 3-19 to 3-21. Air at condition a isin contact with water at three different temperatures in cases 1,2, and 3. In case 1 aa, >t {iq} is from the air to the water sine W, > W, a m>h and since both da, and day, are from the air to the water. In case 2 dq, is from the air to the water since fy > f day, is from the water to the air since W, —_—_— 56 REFRIGERATION AND AIR CONDITIONING y i RN | Re { ¢ SS Ss i ‘ water tenperaie p 1 PS | oem ta? ——e Tame sigue 319 Cue gion a0 pie 120 Ca 2.4 om ow x 5 { “li I 4 eninarmnal fu Figure 3:21 Case 3,4, fiom the water ta. Before the concept of enthalpy potential was developed, we were unable to Gost ‘mine immediately which way magnitudes of da, and day,..Nov In case 3 da, dq, was flowing because we did not know the relative wo the relative values of h 21d h provide the clue. {s from the air to the water since ty > tr dd, is from the water tothe air since My < Mi {2qy, is from the water tothe airsinee ty nd (35), ealulate (a) the humidity ratio if this irs aiabatically saturated, (5) the enthalpy of the air if it is adiabatically saturated, (c) the humidity ratio ofthe sample sing Eg. (35), (d) the partial pressure of water vapor in the sample, and (e) the rela- tive humidity. Ans. (@) 0.0201 ke/kg, (5) 76.2 ki/ke, (c) 0.0180 ke/ke, (4) 2840 Pa, (67%. 3.3 Using humidity ratios from the psychrometric chart, caleulate the error in cor dering the wet-bulb line to be the line of constant enthalpy at the point of 35°C ddsy-bulb temperature and 50 percent relative humidity. Jd An airvapor mixture has a dry-bulb temperature of 30°C and a humidity ratio aHoots. Calculate at two different barometric pressures, 85 and 101 kPa, (a) the Shthalpy and (b) the dew-point temperature. Ans. (a) 68.3 and 68.3 Kis, (8) 17.5 and 20.3°C. 35 A cooling tower is a device that cools a spray of water by passing it through a Jieeam of ai If 15 m?/s of air at 38°C dry-bulb and 24°C wetbulb temperature and sNvaemospherie pressure of 101 KPa enters the tower and the sir leaves saturated at S1°C. (a) to what temperature can this airstream cool a spray of water entering at 3a°C with 0 flow rate of 20 kg/s and (b) how many kilograms per second of makeup ater must be added to compensate for the water that is evaportted? Ans. (a) 31.3°C, (0) 0.245 els 36 In an airconditioning unit 3.5 m3/s of air at 27°C dry-bult temperature, 50 per aac relative humidity, and standard atmospheric pressure enters the unit. The leaving SShaition of the airs 13°C dry-bulb temperature and 90 percent relative humidity. Uning properties from the psychrometrc chart, (o) calculate the refrigerating capacity in kilowatts and (8) determine the rate of water removal from the air. Ans. (a) 88 kW, (@) 0.0113 kels. 5:7 A stream of outdoor air is mixed with a stream of return airin an air-conditioning arrtcmn that operates at 101 kPa pressure. The flow rate of outdoor aris 2 ke/s and its cretion is 35°C dry-bulb temperature and 25°C wet-bulb temperature, The flow rate = Sfiretumn air is 3 ke/s, and its condition is 24°C and 50 percent relative humidity. = Botermine (a) the enthalpy of the mixture, (2) the humidity ratio of the mixture, (e) the dry-bulb temperature of the mixture from the properties determined in parts (@ and (0), and (2) the dry-bulb temperature by weighted average of the dry-bulb temperatures of the entering streams. Ans. (2) 59.1 ki/ks, (8) 0.01198 ke/ks, (¢) 28.6°C, d) 28.4°C. 38 The air conditions at the intake of an alr compressor are 28°C, $0 percent relative humidity, and 101 KPa. The air is compressed to 400 kPa, then sent to an intercooler. If condensation of water vapor from the compressed air is to be prevented, what is the tninimum temperature to which the air can be cooled in the intercooler? Ans. 40.3°C. 3.9 A winter air-conditioning system adds for humidification 0.0025 ke/s of saturated steam at 101 KPa pressure to an airflow of 0.36 ke/s. The airs initially at a tempera- ture of 15°C with a relative humidity of 20 percent, What are the dry- and wet-bulb temperatures of the air leaving the humidifier? Ans. 16.0 and 138°C. 3:40 Determine for the three cases listed below the magnitude in watts and the direction of transfer of sensible heat (using Eq, (3-8)), latent heat (using Eq. (3-9)], fand total heat {using Eq, (3-14)]. The area is 0.15 m* and h,= 30 W/m?-K. Air at 30°C and 50 percent relative humidity is in contact with water that is at a tempera~ ture of (@) 13°C, (B) 20°C, and (¢) 28°C. Ans. (a) ~16.5, -423,-120.4 Wi (0) 45.0, 15.1, -29.6 W; (6) -9.0, 116.5, 113.8 W, ———_ 58 REFRIGERATION AND AIR CONDITIONING REFERENCES sien, Wh: Rational Payee Forme, ASME me wl 33.1, 1008041 Ra Pom of Liu into a Ga, rs. ASME, vl 97551977 g ii 1367, September 1933. 968. WA The Evaporation of «Lig info» Gat-A Coesin, Meck ‘Eng, vol. 55, 1567, Septem pes for Ale Conditioning Pact,” Inlet Ines NOW York, (CHAPTER FOUR HEATING- AND COOLING-LOAD CALCULATIONS 41 Introduction Buildings are built to provide a safe and comfortable internal en- vironment despite variations in external conditions. The extent to which the desired intesior conditions can be economically maintained is one important measure of the ‘success ofa building design. Although control of inside conditions is usually atributed to the active heating and cooling system, the design of heating, ventilating, and air ‘conditioning (HVAC) must start with an examination of the thermal characteristics of ‘the envelope. They influence both the equipment capacity and the energy required for its operation, ‘The primary intent of this chapter is to examine procedures for evaluating the im- pact of the thermal characteristics of the building envelope on the design of the HVAC ystems used to maintain comfort. As the objective of the system is to provide com fort, however, itis advisable to begin with a brief discussion of the factors which in- fluence comfort. 42 Health and comfort criteria "The human body is an amazingly adaptable organism. With long-term conditioning the body can function under quite extreme thermal ‘conditions. Variations in outdoor temperature and humidity, however, often go be- ‘yond the normal limits of adaptability, and it becomes necessary to provide modified Conditions indoors in order to maintain a healthy, comfortable environment. 43 Thermal comfort Figure 4-1 illustrates the factors that influence thermal com- fort. First, body heat is generated by metabolic processes to maintain body tempera- ture. Metabolic processes are influenced by such factors as age, health, and level of 60 REFRIGERATION AND AIR CONDITH Convection nasties) contin —— factor ‘surface temperature Human heat 1oss onING Evaporation “Thermabconort factors igre 4-1 Factor afuencing thermal comfort setvty, For example, a given range of sepals ina space occupied by a healthy Pers ‘but uniceeptable for one who is ill sereptseopte are wing 0 adjust their dres habits ‘with the changing seasons, they find that they are than they would expe "The body is contin constant body temperature. The jomfortable over a br ct Ai motion Radiation Activity Physiological € ne Mar Heath environmental conditions might be quite Trader range of environmental conditions ‘aously generating heat, which must be dissipated to maintain @ carpus mechanisms by which temperate control it consplished were desorbed in Sec. 219 and sr ‘shows in Fig. 4-1. For a person at weet or doing light work in a conditions faees that are at alower temperature Of heat dissipation accounts for API tion, from both 1 Ae environmental conditions or levels For example, if a person ‘anism will be evapora Convection (castied away bY tion. a space, the boty dissipates heat primasily PY “an gurrounding a’) and radiation (to surrounding °° "han the Body surface). Each of these components proximately 30 percent ofthe heat Jos. Eyap0re Mariration and perspiration, accounts for the ‘emaining 40 percent. ier activity change these percentages will Vary onsing strenuous work, the primary heat-dissipation mech > 3 HEATING: AND COOLING-LOAD CALCULATIONS 61 Four environmental factors influence the body's ability to dissipate heat: air temperature, the temperature of the surrounding surfaces, humidity, and air velocity. ‘The amount and type of clothing and the activity levels ofthe occupants interact with these factors. In designing an air-conditioning system we turn our attention to the control of these four factors. Ifa person is wearing appropriate dothing, the following ranges should usually be acceptable:* . Operative temperature. 20 t0 26°C Humidity. A dew-point temperature of 2 to 17°C ‘Average alr velocity. Up t0 0.25 m/s “The operative temperature is approximately the average of the air dry bulb tempera ture and the mean radiant temperature as long as the mean radiant temperature is less than 50°C and the average air velocity is less than 0.4 mys. The mean radiant tempera- ture is the uniform surface temperature of an imaginary black enclosure with which an Secupant would have the same radiant energy exchange as in the actual nonuniform Space. A person wearing heavy clothing may be comfortable a: lower temperatures; conversely, lighter clothing and higher air velocity may provide comfort despite higher temperatures. The temperatures of surrounding surfaces have an influence on comfort fs great as that of the air temperature and can not be neglected. 44 Air quality Air quality must slso be maintained to provide a healthy, comfor table indoor environment, Sources of pollution exist in both the internal and external ‘environment. Indoor air quality is controlled by removal of the contaminant or by Gilution. Ventilation plays an important role in both processes. Ventilation is defined {s supplying air by natural or mechanical means to a space. Normally, ventilation js made up of outdoor air and recirculated air. The outdoor air is provided for dilution. Jn most casts odor and irritation of the upper respiratory tract or eyes are the reason for ventilation rather than the presence of health-threatening contaminants. The pos- sibility of contaminants cannot be overlooked, however Reference 2 prescribes both necessary quantities of ventilation for various types of occupancies and methods of determining the proportions of outside air and te- circulated air. If the level of contaminants in outdoor air exceeds that for minimum sirquality standards, extraordinary measures beyond the scope of this text must be used. For the present discussion it will be presumed that outdoor-air quality is satis: factory for dilution purposes. Table 4-1 presents outdoor air requirements for ventila- tion for three occupancy types listed in the standard. As noted in the table, much larger quantities of air are required for dilution in areas where smoking is permitted. ‘Veatilation imposes a significant load on heating and cooling equipment and thus is a major contribution to energy use. Space occupancies and the choice of ventilation rates should be considered carefully. For example, if smoking is permitted in part of a ‘building but restricted in another part of the building, ventilation rates for smoking should not be assumed uniformly. Also, the prospect of filtering and cleaning ait for recirculation must be examined carefully. The use of recirculated air will conserve 62 REFRIGERATION AND AIR CONDITIONING ‘Table 4-1 Outdoorair requirements for ventilation - Ontdorar semen’ penn, Lis imate goupuncy omnes 7 0 2s Weigand walingspact 6D ins 3s tent $0 23 23 penbien energy whenever the outdoorair temperature is extremely high or ow. The ASHRAE ‘Standard? provides the following procedure for determining the allowable rate for recirculation Pei, where rate of supply ac for ventilation purposes, Us J, =recrealtion aerate, Lis Fore outdoors tat for specie cxpany, for example th non sane valu fom Table 41-but never than 25 (sper person ¥- E where V, = outdoor-air rate from Table 4-1 for specified occupancy (smoking or non- smoking, a appropriate), Lis _E=elfcieney of contaminant removal by airsleaning device, The efiiency fhust be determined relative to the contarinant o be removed. Table 42 provides values appropriate for removal of Im particles also Example 41. Determine the ventilation rate, outdooralr rate and rectculatedsit tate for an office-bulding meeting room if smoking is permitted. An aircleaning ‘Device with £= 60 percent for removal of tobacco smoke is available. ‘Table 42 ASHRAE dust spot efficiencies (-um particles)? iter type Eiticieney range, % ‘Application 525 ust ad lint removal 25-40 Same a above and fr some indus application Mats of 3 t0 104m 40-80 Bulldingrecuculated- fiber 6 0 20 mm thick feahale systems Mats of 0.5-t0 442m 80-98 Hospital surgeries, clean fiber (usually gas) rooms, special applications Blectostatic (depending 20-50 ellen and aiborne particles on type) HEATING: AND COOLING-LOAD CALCULATIONS 63 wD Solution Table 4-1 indicates that 17.5 L/s of outdoor air per person would be re- ‘uired to ventilate the space without any secirculation and ar cleaning. The table also indicates that 3.5 L/s per person is the required outdoor-ir rate for non- smoking spaces and may be assumed, for purposes of th example, to be the ‘minimum rate. There are two possible solutions for this design problem: (a) ‘supply 17.5 Ls of outdoor air per person or (6) calculate the allowable recircula tion rate and corresponding required ventilation rate as follows: 178- ‘607100 Then V= 23.3 + 3.5 = 26.8 L/s per person. ‘Although the total ventilation rate is higher for the second approach in Example 4-1, the energy requirements may be less due to tue reduced outside-air flow rate. If contamination such as tobacco smoke, body odor, moisture, or a high CO con- tent is the result of occupancy, ventilation is not required when the space is not ‘occupied. If other sources of contamination exist, however, such as equipment or processes, outgassing from materials, or naturally occurring preduction of radon, an appropriate level of ventilation must be maintained even if the space is unoccupied. Each of the factors influencing comfort must be kept in mind inthe design of an airconditioning system, These factors have an impact on system capacity, system ‘control, and the design and placement of the duct system or terminal units. For ex- ample, placing heating units under a window or along an exterior wall may offset the effects of the lower temperature of those surfaces. 2 =233Ls 45 Estimating heat loss and heat gain Heat transfer through a building envelope is in- fluenced by the materials used; by geometric factors such as size, shape, and orienta- tion; by the existence of internal heat sources; and by climatic factors. System design \= requires each of these factors to be examined and the impact of their interactions to be carefully evaluated. ‘The primary function of heatloss and heatgain calculations is to estimate the capacity that will be required for the various heating and airconditioning components necessary to maintain comfort within a space. These calculations are therefore based fon peak-load conditions for heating and cooling and correspond to environmental conditions which are near the extremes normally encountered. Standard outside de- sign values of temperature, humidity, and solar intensities are usually available from handbooks. ‘A umber of load-calculation procedures have been developed over the years. ‘Those developed by ASHRAE® will be used here. Although other procedures differ in some respects, they are all based on a systematic evaluation of the components of heat loss and heat guin, Loads aro generally divided into the following four categories ig. 42): ‘Transmission, Heat loss or heat gain due to a temperature difference across a building clement 64. REFRIGERATION AND AIR CONDITIONING filtration nema om tf] | Tafitration igure 4.2 Categories of esting and cooling ods ‘Sola, Heat gun due to transmistion of slar energy though transparent building “omponent or absorption by an opaque building ComPoren! “Inflation. eat loss of heat gain due to the infiltrador ‘of outside air into a condi tioned space “Internal. Heat gain due to the release of energy within »sPace Cights, people, equip ment, etc) In response to these Jouds the temperature in the space #1 change or the heating oF veer equipment will operate to maintain a desired wpe In the following seragraphs we outline procedures for evaluating cach cf these load components. A repre detailed presentation is avallable in Ref. 4. & 446 Design conditions The design conditions usualy sreied [t estimating heating 46 pei nade and outade dry bulb temperatures. For hese ‘operation an in: loads ar cure of 20 to 22°C is generally assumed, and for soo} ‘operation 24 {0 se eypea. A minimum relative humidity of 30% inthe win ‘and a maximum of aoe ie the summer is ao assumed. For heating operon the 975 percent value of See outside temperature is usually chosen. This means {83k Jong-term basis the te ere day bu temperature equals or exceeds this valve for 975 percent of the puts uring the coldest months of the year. At the 975 perent ‘outdoor temperature the ai is assumed to be saturated. a eer of conditions speciied for coolingtoad estimates Tor ‘complex and inchute diy bulb temperature, humidity, and solar intensity. Peak-load conditions vvurng the cooling season usually correspond to the Tami ‘solar conditions rather during nc peakoutdoorair temperature. Thu, i often nesessty 1 make several_ ta eons at diferent times of the day or times of the year f° fix the appropriate & HEATING: AND COOLING-LOAD CALCULATIONS 65 ‘maximum-cooling-apacity requirements. When the coolingoad calculation is made will depend on the geographic location and-on the orientaon of the space being considered. For example, peak solar loading on an east-facing room may occur at 8 A.M, while for a west room the maximum load may occur at 4 P.M. Peak solar loads for south-facing rooms will occur during the winter rather than the summer. Of course, when a cooling system serves several spaces with different orientations, the peak sys. tem load may occur at a time other than the peak for any of the several spaces. For tunately, after making a number of such calculations one begins to recognize ikely choices for times when the peak load may occur. ‘Table 4-3 provides outdoor design temperature data for a number of locations. The table provides the 97.5 percent dry-bulb temperature for winter and the 2.5 per cent dry-bulb and coincident wet-ulb temperature for summer. The 2.5 percent dry- ‘Table 43 Design temperature data Winter Summer 2.5% ary bub) ‘soinedeng wet ‘August daly cy 97.5% dry bulb, °C ‘bulb, "C average, °C Albuquerque, N. Mex. 33/16 “on ‘Adants, Cs 33723 26 aie / 2 sin 2 33/23 2 32/23 2 36924 » 35/15 2 a8 2 Great Falls Bont, site 19 Houston, Tex 0 sans 2 Las Vegas, Nev. al ane 31 os Angels, Cali. ‘ sap. 2 Memphis, Tenn, -8 spa n ‘Miami, Fa 8 3ap2s 28 Minaespolis, Minn. 4 3773 2 ‘New Orleans, La =4 33726 2% New York, NY. 9 32723 2» Phoenix, Ariz 1 ana 3 Pitsbargh, Pa a4 31/22 2 Portland, Ores m4 30/20 20 Sacramento, Cali ° 37a. 2% Salt Lake City, Utah a3 35/17 2% San Francisco, Cai, ‘ 27 7 Seattle, Wash, “3 289 18 ‘Spokane, Wash a7 337 20 St.Louis, Mo. a3 347 25 Washington, DC. -8 35723 25 (66 REFRIGERATION AND AIR CONDITIONING . bulb temperature isthe temperature exceeded by 2S percent ofthe hours during June ralseptember. The mean coincident wet-bul temperature isthe mean wet ob > feature ocerting at that 25 percent dry bul tempersture, Tables 4-10 19 41°. which Peride additonal data relative to the solr lad, wil be disused when the sl load er windows and the thermal transmission for walls and roof ae studied. Example 42 Select ouside and inside design temperatures fora building 40 Pe constructed in Denver, Colorado. Solution From Table 43 the summer design conditions ae given a8 ‘Summer design dry-bulb temperature = 33°C Coincident wetbulb temperature = 15°C ‘Assuming that no special requirments exist, an inside desl temperature 9f aie relative humidity i chosen, The winter desig outside Cemperatre 2 UC irom Table 43, and if no special requirements exist an inside desien ‘eunperatute of 20°C and 30% relative humidity is chosen. it should be noted thatthe inside design temperature only limits the conditions that gare maintained in extreme weather. During heating operation, when the 00 that can Mature i above the ouside design value, an inside temperature greater han 20°C can be maintained if desired. 447 ‘Thermal transmission The general procedure for calelating heat loss or hea in by thermal transmission is to apply Eq. (2-12) At, UAC, -1) ot UIRiop WIK = total thermal resistance, K/W j= overall heattransfer coetTicient W/m? = K (A= surface area, m? tg ~t;= oulside inside temperature diferenee, K 22 neltng load estimates the temperature difference is simply the 97-5 percent outside value minus the inside design value i assed in Chap. 2 the overall heat-ransfer coeficent Us a fonction of ee eetSgces Table 44 (p68) provides values of thermal resistance applies thermal sr urface for commonly Used building materials, enclosed alr space) ant 1 cope boundarits. Example 43 iutates deermnation ofthe U vals of ipa wal erss section. The area used inthe transmission caleslatons ae ‘nominal inside areas of the spaces. Example 43 Determine the total thermal resistance of « unit are of the wall section shown in Fig. 43. HEATING: AND COOLING-LOAD CALCULATIONS 67 — Outside sim Face brick, 90 mit Air space Sheathing, 13-mse Sberboard Insulation, 75-mm mineral ber Air space ‘Gypsum board, 13 mm ee igure 43 Wall section In Example 43. If below.grade spaces are not conditioned, the heat Joss through below-grade surfaces is often neglected. Heating loads are included in such cases based on an esti- spate of the temperature of these unconditioned spaces and transmission through the floor above them. If the belowgrade spaces are to be conditioned, transmission heat Jovees are based on the wall and floor thermal resistance, the inside temperature to be maintained, and an estimate ofthe ground temperature adjacent to the surface. For slab-on-grade construction the heat loss is more nearly propértional to the tength ofthe perimeter ofthe slab (in meters) than its area, Thus Ganay *Floerimetes) (fy 4) whese F= const Little information is available on which to base values of F fr large slabs. Values for residentialscale slabs are given® as F = 1.4 W/m + K for an uninsulated edge and F = (0.9. Wim = K fora slab with 2.5 cm of insulation at the ed. These values must be viewed as approximate and aro generally considered too high, 44 Infiltration and ventilation loads The entry of outside air into the space in- uences both the air temperature and the humidity level in the space. Usually a istinction is made between the two effects, referring to the temperature effect as sensible lood and the humidity effect as latent load. This terminology applies to the ‘ther load components as well. Fr example, transmission an¢ solar loads are sensible, ts they affect only temperature, while internal loads arising from occupancy have both Sensible and latent components. Heat loss or heat gain due to the entry of outside alt is then expressed as Ge 12305-49300, - W,) where Q = volumetric flow rate of outside air, L/s W= humidity ratio, water to air, ke/kg Infiltration, defined as the uncontrolled entry of unconditioned outside air directly into the building, results from natural forces, eg., wind and buoyancy due to the temperature difference between inside and outside. For our purposes we define ventilation as air intentionally brought into the building by mechanical means. OF 0 ‘Table 4-4 Thermal resistance of unit areas of selected building materials at 24°C” ‘mean temperature & = Uksm= KW Rsk” Exterior material ace brick 0.6 0,068 : Common brick 13 7 | Stone 03s 5 ‘Concete block, sind and gravel agrezate, 200 mm ous : ‘Lightweight sggrerate, 200 mm 038 : 150mm 029 ‘Stucco 1s : ‘Siding, asbestorcement, 6 mm, lapped 0.04 . "Aaphalt insulating, 13 mm 014 Wood plywood, 10mm 010 ‘Aluminum or sec, backed with ox insulting boasd, 10 mm ‘Sheathing [Asbestoncement 13 Plywood 66 Fiterboard, guar density, 13 mm 023 Hardboard, medium density 249 Particle bot medium density 138 Roofing ‘Asphalt shingles 0.08 Concrete ‘Sand and gravel agareeate oss Lightweight apzrenate 194 Insulating materials Blanket and bat, mineral fiber, 75-90 mm 194 135-168 mn 335 ‘Board and slab, lass fiber, organi bond na ‘Expanded polystyrene, extruded m7 Collar polyurethane 8 ‘Loos fil mineral ber, 160 mm 338 Calllosc 217-286 “Interior materials {Gypsum ox plaster board, 15 mm 0.08 16 mm 010 Phster materials, cement plaster 139 ‘Gypaum paste, lightweight, 16 mm 0066 ‘Wood, sot (fi, pine, ete) 866 Hardwood (maple, oak, et) 6a ‘Table 4.4 Thermal resistance of unit areas of selected building materials at 24°C mean BD temperature (cont.) temperature (ont) ~ km KW Rim KYW" ee - Air redtance rr 2 Surface, sil air urace emissivity of (09) horizontal, heat flow up Horizontal het flow down Vertical, heat flow horizontal Surface, mong air, heting season, 6.7 ms cooling season, 3.4 m/s ‘Airspace, surface emissivity of 0.8, hortront Vertical Surface emissivity of 0.2, horizontal Vertial Nett ‘lt glo Co Y, Win? * Kt Summer Winter err Single alas 59 62 Double lass, 6mm alr space 33 33 13am ai space 32 28 “Trip las, 6mm ae spaces 23 22 Timm ar spaces 22 18 Storm windows, 25 to 100-mm als space 2 23 {Includes inside and outside ar film resistance Solution The following resistances are obtained from Table 4-4 Outside sir fm 0.029 mK) Face brick, 90 mm 0.068 Air space 0470 Sheathing, 12-mm fiberboard 0.232 Insulation, 7S-m mineral fiber 1.940, ‘A apace 0.170 ‘Gyprum board, 13 mm Inside ae lm R course, the air entering must also leave by natural means, i.c, exfitration, or be ex- hhausted by mechanical means, In commercial and institutional buildings it is considered advisable to control the ‘entry of outside air to assure proper ventilation and minimize energy use. As infiltra- ton is uncontrolled, these buildings are designed and constructed to limit it as much as ‘possible. This is done by sealing the building envelope where possible, sing vestibules or revolving doors, of maintaining a pressure within the builing slightly in excess of 70 REFRIGERATION AND AIR CONDITIONING fable 45 Infiltration constants for infiltration in Ea. (41) ‘Quality of construction e ’ e Tint ots ‘oo10 0.007 ‘Average 020 dois 0014 Toor 03s 0.020 0.022 that outside, However, ia building aos not have mechanical ventilation of he fens a atc are not operating, infitation will occur. Te volumetric flow re of in th et is rather ficult to determine with any measure of precision 9t Ay’ with the quality of construction, wind speed and direction, indoor outdoor wae cate afference, and internal pressure in the building One prose that is area in load calclations isto estimate the infiltration in terms of the names of a mngss per hour. One ait change pet hour would bea wolumetse fow rate Nurs tity equal tothe internal volume ofthe space, The mumber ofa hanes PoE hour ically mre building with no internal pressurization can be estimated 252 function’ cof wind velocity and temperature difference ‘Number of air changes =a + V+ c(t, ~ 4) (at) where a, b, ¢= experimentally determined constants id velocity, m/s “Typical values ofa b, and care presented in Table 45, For nonresidential buildings it Typical val to uas estimates of infiltration for load calculations only when the Tone 2 aeetatlationaystem are not operating, An example would be for izing 3 Posing & ern to malntin a minimum temperature during an unoecupid shoe period. Co ee netic flow rate of outside at fr ventlation is competed from Tsbie 8 and ry ettods outlined in Section 44. The lightly posive presue inthe Puli and the ned by alzing the exhaust fans fo handle lst ait than brought jn om the 2 apy the venation system. Also, exhaust fans are genealy cated in restvoot aac eal rooms, or kitchens to ensure that air and odors fram thew space will mechan culated throughout the Building, Chapters 5 and 6 will rovide move WOE ot Be eae design of the airistibution system. It shoud be noted st (hs Polt mation on en although the outdoor component of ventlation imposes » oad 8 omtng ind coating equipment, the load occurs at the point where i conditioned te ae inthe space. It is therefore necessary to distinguish between canines TEINS ghd the space Toads used to determine the airflow requted forthe ‘building spaces. 145° Summary of procedure for estimating Heating lods In etimating oe hear te a piling, tis important to use an organized, stepbystep Procedure ‘The novessary steps can be outlined as follows: 1, Select design values for outdoor winter design (97-5 perent vale) from Table 43. wo HEATING: AND COOLING-LOAD CALCULATIONS 71 2, Select an indoor design temperature appropriate to the actvilies to be carried out in the space and a minimum acceptable relative humidity. 43, Determine whether any special conditions will exist, such as adjacent unconditioned spaces, Estimate temperatures in the unconditioned spaces as necessary. 4, On the basis of building plans and specifications, calculate heat-transfer coefficients ‘and areas for the building components in each enclosing surface. Any surfaces . ‘conaecting with spaces to be maintained at the same temperature may be omitted, ives interior walls. 5. On the basis of building components, system design and operation, wind velocity, and indoor-outdoor temperature difference, estimate the rate of infiltration and/or ventilation outside air. Note that the latent component of the infiltration and/or ventilation load is included only if the conditioned air is to be humidified to main- tain a specified minimum indoor humidity level. Humidificaion is often omitted from air-conditioning systems in mild climates. 6. Using the above design data, compute transmission heat losses for each surface of the building envelope and the heat loss from infiltration an¢/or ventilation. Sum these values to determine the total estimated heat loss and the required capacity of the heating equipment. 7. Consider any special circumstances that might intiuence equipment sizing. Three ‘igcumstances may influence equipment capacity Ita building and its heating system are designed to take advantage of passive solar gain and thermal storage, heating capacities should be based on dynamic rather than static heat-loss analysis (see Chap. 20) b. Ina building that has an appreciable steady internal load (heat release) at the time of the maximum transmission and ventilation heat los, heating equipment capacity may be reduced by the amount ofthe internal heat release. An example ‘would be a hospital or industrial building which operates on a 24+ basis. A building that does not operate on a continuous basis and indoor temperatures ae allowed to drop over @ lengthy unoccupied period, additional capacity may be required to bring the sir temperature and building indoor surface tempera tures back to an acceptable evel ina short time, An alternative to the additional Capacity is to bring the heating system into operation earlier and allow the ‘building to heat more gradually. 4-10 Components of the cooling load Estimating the cooling load is more complex than estimating the heating load. Additional consideration must be given to internal loads, latent loads, and solar loads. 411 Internal loads The primary sources of internal heat gain are lights, occupants, ‘and equipment operating within the space. Internal loads are a major factor in most ‘nonresidential buildings. The amount of heat gain in the space due to lighting depends fon the wattage of the lamps and the type of fixture. When fluorescent lighting is used, ‘the energy dissipated by the ballast must also be included in the internal load. As light- {ng is often the largest single component of the internal load, care must be exercised in its evaluation. The portion of the heat emanating from lighting which is in the form of —————— 72. REFRIGERATION AND AIR CONDITIONING radiant energy is not an instantaneous load on the airconditioning system. The radiant aes) om the Lights is first absorbed by the walls flor, an¢ furnishings of the Space and their temperatures then increase a arate dependent ‘their mass. AS the SPtface temperature of these objects rises above the air temperature, heat is convected surface tembfaes and finaly becomes a Toad on the cooling stem. Thos betes of from the sine objects absorbing the radiation there i dehy between ‘woe the Hight on and the energy from the ights having a effect on Oe oa ‘The cooling load teen She lighting persists after the lights are turned off forthe same Pali To ac: fiom te thee cucumsances the following format has teen developed for esti- mating the internal heat gain from lights* = (lamp rating in watts) (F,) F,) (CLF) utilization factor or fraction of installed lamps in ust ethat ret for Mrescent lamps = 1.2 for most common fiuorescent fixtures CLE = coolingJoad factor from Table 46 ‘table 4 pronder coolingload factors for two common Fxture arrangements and where Fy Ps ‘Table 46 Coolingload factors for lighting Fixture Xt. inture Yt No, of hours routs of operation ‘ours of operation after ights = — ae tured on 1016 0 16 ° 008019 oor 005 1 0620.72 076 079 2 066 (O78 pal 088 3 069 (O17 oss 087 4 0730.80 oss 089 5 075 082 090091 6 078 O84 092 093 1 oso 085 093094 a 087 09s 095 088. 0.96 089 097 030 098 ot 098 092 098 093 099 094 039 094 099 040 024 036 020 "which are not vented. The supply and rt Sing or though the calling space aad eile. Y, vented oF ‘hvough ihe cling withthe xe da20300. oeedecce ey HEATING: AND COOLING-LOAD CALCULATIONS 73, ‘Table 4-7. Heat gain from occupants Sensible Activity Heat gain, W heat gin % is /— ee Sleping 70 15 Seated, quiet, 100 6 a Standing 150 50 ‘Watkng, 3 km/h 30s 35 Office work 150 5s ‘Teaching 115 50 Reta shop 188 50 Industri 300-600 38 eit CU TT lights operating 10 and 16 h/d. Additional information covering v tures, floor mass, and operating periods is available." For heat-producing equipment it is also necessary to estimate the powe: used along with the period andjor frequency of use in a manner similar to that used for lighting. For equipment having litle radiantenergy transmission the CLF can be as sumed equal to 1.0. ‘Table 4-7 shows loads from occupants asa function of their activity. The greatest ‘uncertainty in estimating this load component isthe number of eupants. If the nur ber of ovcupants is unknown, values such as those in Table 46 may be used. Since 3 portion of the heat transfered from occupants is by radiation, the ASHRAE method- Glogy again uses the coolingload factor for a better represenlation of actual loads Table 4-9 gives these values. Thus, Occupant sensible cooling loadin watts ain per person from Table 4-7 X number of people X CLF from Table 4-9 For the latent load the CLF is 1.0. ‘Although there are # number of uncertainties in estimating internal loads, these toads ate significant and must be evaluated as carefully as posite 412 Solar loads through transparent surfaces Heat gain due to solar energy incident ‘on a surface will depend upon the physical characteristics of the surface. Surface op- ‘Table 48 Space per occupant? ‘Type of space Occupancy Residence 246 occupants Office 10-15 m? per occupant Retail 3-5 m2 per occupant School 2.5 m2 per occupant ‘Aditoriam 11.0 m? per occupant 74 REFRIGERATION AND AIR CONDITIONING ‘Table 4.9 Sensible-heat cooling load factors for people’ Hours after “Total nour in space cach entry intoapace 2 ; es 0 m 1 pap oa9 050 «O51 «083 OSS 058 O62 2 Ose 059 (060 061 «(06264 070 3 017066 «(067067069070 07s ‘ 43 O71 072072074 OS 079 5 010 0270.76 (076 07.8 082 6 os oat 07908080 OBL oss, 7 007 016 (034 082083 OBE 8 006 O14 (026084 08S .86 88 9 os 011 021038087088 0.90 10 bos O10 OB 030089089 oot a oot 008 OS 025042 OSL 092 2 003007 043021038092 093, B 003 006 «Olt 048028 OS 094 1“ 002 006 0101S 023036 03s 1s cz 008 0.08 013,020 030 095 16 02 004 = «0.0702 OT 025 096 0 002 00¢ 005 OAD OS OT 049 8 or 003005009 OA3—OD 039 Figure 44 Dist sbution of sola heat stuking a transparent sft. neechere HEATING: AND COOLING-LOAD CALCULATIONS 75 tical properties are described by rtpta ‘where + = transmittance p= reflectance a= absorptance zi ‘The value of each of these terms has a pronounced effect on sola-heat gain. FPor transparent surfaces, such as the window shown in Fig, 44, the solar energy passing through the surface dyg in watts is yg 7 All, + Na) = Al (+ Na) 42) where [, = itradiation on exterior surface, W/m? faction of absorbed radiation transferred by conduction and convection to inside environment hy = outside heat-transfer coefficient, W/m? + K Under sweady state conditions N ean be shown to be Ujhg. Restating the equation in ot and hy ee we tana (+) ‘The expression /,(r + Ualh,) for a single sheet of clear window glass is frequently feed to as the solarheat gain factor (SHGF). Maximum values* for the SHGF are sven for two latitudes by month and orientation in Table 4-10. ‘Table 4-10 Maximum solarheat gain factor for sunlit glass" W/m? Nitade NEW EW SEISWS Hor 32" north latte Dee o 0 310 ms 795 (S00 Jan, Nov 18 90 350 ms IS SSS Feb, Oct 8s 205 as 70 © 1008S Ma, Sept 100 330 9 jo s45 780 ‘Apr, AU us 450 ‘700 580355 AS May, July 120 330 6s 40030865 June | 140 555 61s 440190870 40° north iattude Dee 37 a as 70 «8008S Jan, Nov oS 3 480 ss 9S 20 eb, Oct 30 155 315 760750, (565 Mar, Sept 95 285 660 m0 4090 Apr Ate 110 435 690 eo as 790 May, July 120 sis 690 Sas 350830 June 150 540 680 S10 30040 Sues 16 REFRIGERATION AND AIR CONDITIONING Table 4-11. Shading coefficients - ‘Shading Coefficient ‘Venetian binds Roller shades ‘Thickness, No Indoor —— “Type of glass mm ‘Shading, Media —‘Light_—Dark ‘Light Single eas Regular sheet 3 1.00 oss oss 059025 Piste on 09s Oss 05S 0890.28. Hestabsorbing 6 070 057053 0400.30 10 050 oss 052040028 Doble lass Regular sheet 3 090 0s7 0st 0600.28 Phate 6 083 osT 0s] 060028 Reflective 6 02-04 02-033 ‘shading coefficient ($C) is used to adjust these SHGF values fr other types of alas orto account for inside shading devices, This coset is 14 Ualh, sow 21 Ualte GF Gall, where the subscript ss stands for a single sheet of clear lass, Typical values of the weet coefficient for several types of glass with and witheut internal shading are Pre aan Table #11, If external surfaces shade the window, SHGF values fo a north entation ate used forthe shaded portion ofthe window. “The colar energy passing through a window dy, can be expressed 28 yy ~ SHGF gy) (SOA ‘One more factor must be considered since the solar energy entering the space does not appear instantaneously a2 a load on the cooling system. The radiant ener i st psorbed by the surfaces in the space, during which time these surface temperatutes aes at rate dependent on their dynamic thermal characteristics. Thus, the solar nergy absorbed Is delayed before being transferred tothe alr in the space by conic Srere since this process may involve a significant time lag, itis also usual practice 19 sete « eooting load facor (CLF) in calculating the cook loud attributable 0 Fai. sac oush gst, Values of CLF derived from an exteasive computer analysis are presented in Table 4-12. ‘When estimating solarheat gain through’ transparent surfaces, external shading aust be considered. Shading from overhangs or other projections, such as shown in Fig, 4-5, cam sigiicantly reduce solaheat gain through & window. The 4ep 6) re sa cast by 2 horizontal projection above a window can be calculated using he snedoTfotude angle fand the wall-azimuth angle 7, where is the angle measured fron) «oi anal plane on earth wp to the sun and 7 i the angle between two vertical HEATING: AND COOLING-LOAD CALCULATIONS 77 ‘Table 412 Cooling oad factors for sass with interior shading, noth latitudes’ ‘Window facing Sour Se oN Me Ese SSW OW ONW How Gers 056-047-030: 009 0.07 0.06.07 0.17 § os 076 «0.72037 04G OAL 0.09 OAL 7 Ges om 080 074 0.23 OMe oe 073 058 076 «O81 0380.16 3047 3 ko 037062 079058 0904S O07? 10 oes 029 OdL 08807502 ONG 0.20 OL ges 027027 049083038 O17 OT OS BR ga 026 0.24 033 «080 05903 0.22 OS Bee 0280.22, 028068 0.75 053-030 OT Mo as oz2 0.20025 050083072052 OL 1s Oss 020017022035 «OSL 0820.73 OS tre ode Odd 018027 09 O81 O82 042 yo Gor oaz Oss 013 als o4s O61 690.28 Figue 45 Shading angles and mensions ee 78 REFRIGERATION AND AIR COND(TIONING ‘Table 413 Solar poston angles for the twenty-first day of month? Solar time, AM. wen we 32° north atitude : 8S Dee 8 1» ° oom oH anor 8 1 Bb BM % ° 6% &% 3 Foot F ye eT ° np @ 8 8 Uo Mazsep 8 Bow 7 47 3s 88 ° 2 3 2 7 7m oO Api Aug 8 6s op on mw 6 GSO ° 10k DT MayJl 8 8s LB ¢ 1710093 BO jun 8 > mea so boos 7 40" north tttude 23 0 Dee ’ ° so sO JanNov 8 B17 me HD ° sow 3 Feboct |B ao MRT ¢ nh 2 0 3 Bo Mar,sep 8 ho op ms @ 4 8 ° o 0 37 &@ Bo aps, Aue 8 > 3 SS ° o 8 Bp 7 1 Bo Mayol 8 > Bb mw 3s 4 87 GD Ss ea Jun ’ SM eB eT ee 7 es US Ut Solar ime, Pa. eine planes one normal tothe wall and the other containing the son. TAS solar altitude 6 Pane esr arimuth ange 6, which is the ange between the sun's rays a south, aaa in Table 413 for 32° and 40° north latitude, The wllszinoth angle can ‘be computed from the equation 168d where y is the angle a vertical plane normal tothe wall makes with the south ‘The HEATING: AND COOLING-LOAD CALCULATIONS 79 depth ofa shadow y below a horizontal projection of width digiven by ‘The width of a shadow cast by a vertical projection of depth d is xedtany ‘The SHGF and CLF for a north-facing window (in the northem hemisphere) are used for that portion of the window surface which is shaded. Example 4-4 A 1.25-m high by 25-m wide window is inet from the face ofthe ‘wall 0.15 m. Calculate the shading provided by the inset et 2 P.M. sun time ifthe ‘window is facing south at 32° north latitude, August 21 and y= @. From Table 4-13, B= 56° Solution For a south-facing window, and y= 60°; then x=d tan 7=0.15 tan 60° = 0.26 m tang _ 0.15 tan 56° pug nh OES cosy cos 60° 044m ‘Sunlit area = (2.5 ~ 0.26) (1.25 -0.44)= 1.81 m? surfaces The process of solar-heat gain for an opaque wall 4.13 Solar loads on opaque: Fig. 46. A portion of the solar energy is reflected and js illustrated schematically in Conducted Reflected Figure 46 Solar loads on opaque surfaces. 80. REFRIGERATION AND AIR CONDITIONING the remainder absorbed. Of the energy abso tet fonder of te absorbed sla energy transmited the {haide by conduction or temporarily stored "The transmissivity 7 of an opaque surface ated to the outside. hed some is convected and some reradi {s zero and thus for walls and roofs ptard ‘Equation (4-2) therefore reduces £0 1f the transmission due to ua, ay", 1A ane airtemperaturediferene is ircuded, 43) Ue 4g = ETAT UA Equation (43) can then be rearranged 10 Be u,a[ (+2 (4) a6 BA| (0) os sem et appre a h fi rmin bec ened Fr en temperate fy WHETE ip tld Bq, (4-4) can be rewritten as ‘The equivalent temperat ‘ure increased by an amount £0 2c Using the solar temperature opaque surfaces. For opaque walls, quite pronounced, and using aerate the heat gain. To incorporate th ot Efaled the cooling load temperature die perature differ Beveloped® for commonly used wall flux on the surface and fix eral wall and roof sections ae presented Tables plete tabulations will b ets following the tables must be observed fo ‘iffer from the ones on ‘will illustrate this process. ‘The need for including the transient o Here the heat flux theo masses is plotted vers¥s Tower-mass wall is high wall. 497 UWA) called the sobairtemperaure isthe outdoor tempers ‘count for the solar radiation. cerceaventent way of including solar loads for however, the effec’ of thermal storage can DF the temperature difference f= ff MAY significantly over- effect of thermal storage an equivalent tem srence (CLD), has bee rae, sections. It takes into account both the solar the mass of the vall. CLD values “4-14 and 4-15. More com oii Refs and 6. In wing Tables 4-14 and 41° the modify the tabular values for cases that my ealeuation of the CLTDs was based, Example 5 the thermal capacitance of fects is iustrated by the data in Fig, 47. cnt Tyo walls with de sume Uva but seniicanty different nh two Wf day It can be seer thatthe peak Ret so" the the iseurs easier in the day than that ofthe highersmass HEATING: AND COOLING-LOAD CALCULATIONS 81 ‘When the thermal storage is included, the heat gain through the wall is given by y= VACETD) @s) [Example 4 Determine the peak heat gain through a westfacing brick veneer ‘wall (similar in cross section to that shown in Example 4-3), July 21 at 40° north latitude. The inside temperature is 25°C, and the average daily temperature is 30°C. ‘Table 4-14 Coolingoad temperature difference for flat roofs,’ K Mase per Heat Solar time Root wait ea, capacity, << <$<__————————— Det kgm? WimesK 7 8 9 10 1 1243 14 IS 16 17 18 19 20 Roofs without suspended clings 1 35 453 «1:19:27 34 40-43 44 43 39-33-25 17 10 2 40 12-815 2 2935 39 4 41 34 29 3 30 90-2 «1 S11 18 2531 36 39 40 40 37 32 28 i aso 2010 2 4 8 1 18 29 93:35 36 35 32 $350 2304 GB OL 1S 18 22 25 28 29 30 29 27 6 365330 8 7 8 8 1012 15 :8 2022 2 25 26 8s so eee Roofs wth suspended ceilings _eE=—EEE——e——r 1 4s 500 $13 20 28 35 40 43 43 4137 31 23 1S 2 50 8 1 2 4 7 12 17 227 333s 4 32 2B 3 1001002 6 10 16 21 27 HA 36 36 34 30 2 130 13064 4 4 6 912 6 0 24-27 29 3030 $350 2402 AL AL AL 3213.45 46 18 19 20 21 28 @ 3600-40. 13«13.:13.:12-12_-13:13 16 ISt6 16 1718 18 "Noes: 1. Direcdy applicable for the following conditions: ise temperature = 25°C; out side temperate, maximum = 35°C, erage = 29°C; daly range = 12°C; and solar radiation typical Stuy 21 at 40> north latitude, Exact values fr the U value approprae forthe design conditions ‘being considered shouldbe used for calculations Ty Adjstments to these CLTD vales should be made as fllows i the indoor or outdoor d- sien conditions difer from those specified in note CLD yg“ CLID+ 25-1) 4 4-29) inside design dry-bulb temperature, °C erage outdoor dry-balb temperature for design day," 23° For soot gonstiuction not listed choose the rof from the table which it of approximately ‘the same density and heat capacity “rWhen the roof has adaltional insulation, for each R = 1.2 m2 «K/W ination insulation tse the CLTD for the neat thermally heavier f0of. For example, for + type 3 roof with an add tional R = 1.2 insulation, use the type 4 r09f CLID. “F1= shect steel with 25 to 50 rom insulation, 2= 25 mm wood sith 25 mm ination, 3= 100 mmm lightweight concrete, 4" 150 mm lightweight concrete, 5= 100 mm heavyweight com ‘rete, 6= roof terrace system, {82 REFRIGERATION AND AIR CONDITIONING ‘Table 4.15 Coolingload temperature difference for sunt wal ‘Ocenta vast a EES oma, rien typet elm? Kijm?-K Solar time N NE E SE S SW WNW a REESE SS 7 RRR SEG f Ree ita t B FeRbaE EG a ae HoLe gba w oe Bo BN BR BRE B coe e606 BBR BRR BE e BE Gee Bh ® BoB ge as bx eo ue eae oh ar ee fo ERE HB BR BoP RE Be i ee ee rere icra yo be saiai a borg ie aa i B ieee tsa eo tebe ea td BEERS ae Ss B Se RRB ft om om ob SB RB BE MY Hobe Bue eB s RE Rb BR BS a ee a n BERR be Bx HORE ERE R BR S RE RM BB RR cot, BR BM Bb» BX at gh 7 PRRs 3 re ee Boe be tb t 3 e PH aR Sa ts BoTh eh as as B RRB aa es i ~ ow om oR PRR BE EG \ H TEER EM TB BSE Been Be e ee hb bu Bk noe eg ee Re fou Eh Be Ra S PE Ee wk BR at, ERAS be RH > pte HEATING: AND COOLING-LOADCALCULATIONS 83 ‘Table 415 Coolingoad temperature difference for sunlit walls (cont) Mass per Heat Ocentation Wal unitarea, capacity, Yoet Kem?” im2-K Solrtime NONE ESE § SW WNW 7 45 5 46 7 6 8 405 5 4 5 6 57 3 67535 5 4 10 80 73 4 5 4 a w 1 1 4 4 5 4 2 wm os ss 4 B 2M 4 7 6 6 Ss Dp 390380 “ now wm 9 7 6 6 15 Bow 7 mn 9 8 7 16 Bo oH oR 1 8B ” 4 i i 1s 1s 13 10 8 48 8 6 18 17 2 9 4 17 17 % 20 20 15 20 Boy 7 6 2 2 OD CLD, 4 1 6 2 2 8 " 7 67769 0 8 8 6 76 6 8 9 7 9 6875 7 8 6 10 797577 6 nw auo9 5 6 7 § 2 w 3 0 5 6 7 S$ B 4 2 6 6 7 6 cso 480 1 noi 3B a 7 7 6 15 2 Ww m4 9 8 8 6 16 2 1% is ut 9 7 1 Rv 6 22 Uo 18 Bow 3 4 Bo 19 Bo 16 Mi 16 12 20 B16 16 4 18 18 14 CLP nae Bo 16 4 18 2% 15 TNotcss 1. Reference 4 al shows CLTD values for heavier walls suchas 300 mm concrete with tetaton and exterior finish; also 100 men face brick with 50 mm inwulation and 200 rm ws is abl is directly aplicable forthe conditions stated under Note 1 of Table 4-14. 3. ne procedure for corecting for indoor and outdoor temperatures differing from stan. ard is given in Note 2 of Table 4-14. Mal constrsctions not Hsted can be approximated by using the wall with the nearest values of denity and heat capacity. Seals wit edition insulation shit to the wall with next higher mass. wall ype esghated by the preceding eter of the alphabet, fr each additional of 1.2:m?+K/W. For ex- ‘Sale forthe adaition of «1.2 m2-K/M to wall type E, use LTD values fr wall typ D. Be er Gartainor frame wall with 25 to TS mm insilaton.F = 100mm concrete block with 25: $0 mm inulation;or 100 rm face brick wih insulation. E= 200-mim concrete block with in- terior and exterior fini or 100 mmm face brick with 100-mm concrete Mock and interior fish; fr TOOmmn concrete wall with interior and exterior finish. D= 100 mm face brik with 200-mm ‘oncrete block and interior finish; or 100 mm face brick and 100-mm common brick with interior finish. C= 200-mm conerete wall with interior and exterior finish 84 REFRIGERATION AND AIR CONDITIONING 4, = UA(CLTDY § a R 5 ‘Time of ay Figure 4-7 Heat fx through two walls with the same U values but diferent masses Solution The wall most nearly matches type F in Table 4-15. The maximum SIMD scours at 1900 h (7 Pat.) with a value of 33°C. The average outdoor tem perature i 30°C rather than 29°C, on which the table based, so the adjusted CLIDis CLTD=33+ (30 -29)= 34K From Example 43 R = 2.812 m? + K/W, 80 the U= 0.356 Wim? « K. env (CLTD) = 0.356(34) = 12.1 Wim? 4 4:14 Summary of procedures for estimating cooling loads The process of estimating ‘riding loads Is similar to that used in determining heating loads. In fact much of the sont tormation is applicable. There is enough difference, however, to make each step worth noting again. 1. Select design values for outdoor summer dry-bulb temperature (2.5 percent ius), mean coincident wet-bulb temperature, and the dally average temperature from Table 43, 2, Select an indoor design temperature which is appropriate forthe activities to be carried out in the space, , sobs y 33929302 EATING: AND COOLING-LOAD CALCULATIONS 85 . Determine whether any special conditions exist, such as adjacent unconditioned spaces, Estimate temperatures in the adjacent spaces. ‘On the basis of building plans and specifications, compute heat-transfer coeff- ‘Gents for the building components in each onclosing surface. Any surfaces con- fecting with spaces to be maintained at the same temperature may be omitted. Note that the only differences between the U values calculated here and those for the heatingload estimate are the values used for the surface convection coef ‘cients, which differ in summer and winter and may vary with the direction of heat flow. From the building plans and specifications, system operating schedule, and design values of wind velocity and temperature difference estimate the rate of infltra~ tion and/or ventilation of outside air. For the cooling load the latent load is also included. 6. Determine the additional building characteristics, eg, location, orientation, ex ternal shading, and mass, that will influence solar-heat gain. 7. On the basis of building components and design conditions determine the appro- priate coolingload temperature differences, solarheat zain factors, and cooling- load factors 18. On the basis of the heat-ransfer coefficients, areas, and temperature differences {determined above calculate the rate of heat gain to the space. ‘9, For spaces with heat gtin from internal sources (light, equipment, or people), apply the coolingload factor when appropriate. 10, Summ all the pertinent load components to determine the maximum capacity re- ‘quired for heating and cooling. If the building isto be operated intermittently, ‘additional capacity may be required. “The above procedure and the discussion inthis chapter has been bref. The reader is directed to the most receat “ASHRAE Handbook, Fundamentals Volume” ot similar sousces for a more complete discussion of the detalls and for more extensive tabular data PROBLEMS 4-1 ‘The exterior wall of a singlestory office building near Chicago is 3 m high and 15 ‘mong. The wall consists of 100-mm face brick, 40-mm polystyrene insulting board. 150-mm lightweight concrete block, and an interior 16-mn gypsum board. The wall contains three single-lass windows 1.5 m high by 2 m loag. Calculate the heat loss through the wall at design conditions if the inside temperature is 20°C. Ans. 2:91 kW. 42 For the wall and conditions stated in Prob. 4-1 determine the percent reduc- tion in heat loss through the wall if (a) the 40 mm of polystyrene insulation is re- placed with 55 mm of cellular polyurethane, (6) the sindle-lazed windows are ro- placed with double-glazed windows with a 6-mm air space. (c) If you were to choose between modification (a) of (b) to upgrade the thermal resistance of the wall, which ‘would you choose and why? Ans. (a) 12.4% 86 REFRIGERATION AND AIR CONDITIONING. 43 Anoffice in Houston, Texas, t miintained at 25°C and 55 percent relative humid- ity. The average occupancy is five people, and there will be some smoking, Calculate ARE Looting load imposed by ventilation requirements at summer design conditions ith supply ait conditions set at 15°C and 95 percent relative humidity if (a) the seramended rate of outside ventilation ar is used and (b) ia filtration device of E 10 percent is used, Ans. (2) 2.1 kW, (b) 1.31 kW. ‘4A computer room located on the second floor of a fivestory office building is Jo by 7m. The exterior wall is 3.5 m high and 10 m long; i is @ metal curtain wall (greet backed with 10 mm of insulating board), 75 mm of elsefier insulation, and 16 eee of gypsum board. Single-lazed windows make up 30 percent of the exterior wall “The computer and lights in the room operate 24 h/d and havea combined heat release to the space of 2 KW. The indoor temperature is 20°C. ‘EDI the building is located in Columbus, Ohio, determine the heating load at winter design conditions. Ans. 602 W () What would be the load if the windows were double-alzed? “405 Compute the heat gain for a window facing southeast at 32° north latitude Bio ant, central daylight time on August 21. The window is regular double glass with s Tamm air space, The glass and inside draperies hive a combined shading wettrcient of 0.48, The indoor design temperature is 25°C, and the outdoor tempera- ture is 37°C, Window dimensions are 2 m wide and 1.Sm high.Ans. 750 W 46 The window in Prob. 4-5 has an 0.5-m overhang at the top of the window. How far will the shadow extend downward? Ans. 0.55 m. ‘47 Compute the instantaneous heat gain for the window in Prob. 4-5 with the exter- tal shade in Prob. 46, Ans. 558 W. t8 Compute the total heat gain for the south windows of an office building that fas no external shading. The windows are double-lazed with a 6-mm air space and MHth egular plate glass inside and out, Draperies with a shiding coefficient of 0.7 sully closed, Make the calculation for 12 noon in (a) August and (b) December ar 52° north Tatitude. The total window area is 40 m?, Assume that the indgor tem- wrratures are 25 and 20°C and that the outdoor temperatures are 37 and 4°C. Ans. (@) 9930 W, 145 Compute the instantaneous heat gain for the south wall of building at 32° north Matude on July 21. The time is 4 Pat, sun time, The wal is brick veneer and frame dae perl! heat-ransfercoeficient of 0.35 W/m? ~ K. The cimensons of the wall are 25 by Sm. Ans. 87.5 W. 410 Compute the peak instantaneous heat gain per square meter of area for a brick ‘28 wall tmilat to that in Example 4-3, Assume thatthe walls located at 40° north Teitade. The date is July. What time of the day does the reak oocur? The outdoor (ally average temperature of 30°C and indoor design temperature is 25°C. REFERENCES 1. Thermal Environmental Conditions for Human Occupancy, Standird $581, American Society renting, Refigerating, and Air-Conditioning Engnecr, Adanta Ga. 1981. 2S a Venulaton Required for Minimum Accepable Indoot Air Quality, ASHRAE smears 628}, american Society of Heating, Refrigerating, and AirCondtioning Engiecs, ‘Adana, Ga. 1981 pa aaa ys HEATING: AND COOLING-LOAD CALCULATIONS 87 4, “tlandbook and Product icectory, Pipment Volume,” American Secity of Heating, Rei Chtig, and Air-Conditioning Enginers, Atlant, G2. 1979, a Spook, Fundamentals Volume,” American Socety of Heating, Refer, and AirConditoning Engineers, Atlanta, Ga. 196) ease eng Re Achenbach: Fd Measurements of Ai Ifa n Ten Elec ffented Houses, ASHRAE Trans, ol. 69, pp. 358-365, 1963 (eee e caing and esting Load Caeuation Manel” Amescan Society of Heating, Weciporating, and Air Conditioning Engineers, Atlanta, G. 1979. AIR-CONDITIONING SYSTEMS sf. Thermal distribution systems Chapter 4 explained how to compute hesting 5c oads in conditioned spaces. In oder 10 compensate for these fonts svete oon loadsfered to or from the space. In most medium-sized and large bullings ve dane tray is transferred by means of air, water, and occasionally refigersnt- THE deer ereeetagy often requires conveying energy {rom a space toa central heat Sk (Gafageraton unit) or conveying heat from a heat source (eater or bolle) fo 0 See Cotreermmbly that transfers heat between the conditioned spaces and the 390% OF Tee ete the thermal disibution system, Another ofits functions sf inooers aan atilation at The selection of sizes and capac ofthe components fe out ot aera! dstbution systems is covered in Chaps. 6 and 7, respective a aan encsent chapter concentcates on the configuration of the components, We reg ith an explanation ofthe lassie ingle-zone system, which would Bs fora se titorum or 2 laboratory where precise condtans are to be maintainet A sae a potion systems serve multiple zones, The mltple-one systems that wit be studied in this chapter include 1, Airsystems 4. Terminal reheat ». Dual-duct or multizone fe. Variable-ir-volume 2, Water systems a, Tworpipe ». Fourpipe [AIR-CONDITIONING SYSTEMS. 89 Hier Coolingand Heating = enumiifying coll oll o-—_ C ay Return optional Figute 51. A singlezone system, 5.2 Classic single-zone system The elements of the air-conditioning systems that will provide heating (and humidification) or cooling (and dehumidification) are shown in Fig. 5-1. A subsystem of this and most other airconditioning systems controls the flow rate of outdoor ventilation air. This subsystem interfaces with the facility shown in Fig, 5-1 at points A and B. Outdoor-air control is discussed in Sec. 53. From point ‘A the air flows to the cooling coil. heating coil, fan, and hunidifier toward the con- ‘itioned space. In the retum-air line a fan is often installed to avoid excessive air pressure in the conditioned space relative to the outside-air pressure, The temperature control is provided by a thermostat regulating the cooling or heating coil, and the humidity is controlled by a humidistat that regulates the humidifier. Two of the pos- sible operating modes are shown on the psychrometric charts of Fig. 5-2. Figure 5-2a shows & heating and humidification process wherein air at point A having a low tem- perature is warmed in the heating coil and humidified by the direct admission of steam. [As Prob. 39 showed, the process of humidification by direct admission of steam re- sults in little change in dry-bulb temperature. Tecmo : Kooingcot | E a |B 3 —<— 5 Reheat = “79 conditioned soace Temperate "Tempratare © ® Figure 52 (a) Heating and humidification and (b) cooling and dehumiiiation with reheat ———__ 90 REFRIGERATION AND AIR’ CONDITIONING In Fig. §:2b the aris cooled, and ifthe temperate of the metal surface of the at bel the dow point ofthe a, moisture wil conten ‘The heating coil may be Seerated simultaneously withthe cooling ca 9 Wt ithe conbination of the cooling oPartpeat processes provides & steep s0Pe etWeeh 4 and the condition entering the Sac, The combination of reheat with the cooing ‘and dehumidifying process is some space Tod in naalatios where the reatve humidity must ibe kept low or there is an. ‘excessive latent lod. 153 Otdoorsir control The introduction of ovtdor® veniltion air, as discussed in Se enacts necessary when the conditioned spsce js ‘occupied by people. In many set nrconditoning installations the minimum Pores ‘of outdoor ventilation omfortveen 10 and 20 percent of the total flow rae of ‘supply air. In some special cations, es, hospital operating rooms aN, RN for laboratory animals, the srry ae may come exclusively from outdoor 2 te corditioned to maintain the Spesfed space conditions. No return aii recirculated in thes installations. ie Poort contol mechanism that interfaces with air system of Fi amd eines ai aystems presented ater in this chapter i SOW Fig. 53. The stream of aovsen ait at B flowing back ftom the zones dives ‘exhausting and some 12 retatyang The outdoor vetiaton ac mixes with he “ecireslated air and flows to the ie oneag ust at A. Dampers inthe outdoor xR and recirculated-air lines sontite the flow rates. The dampers in the outgeo™ ind exhaustair lines open and rein unison and in the direction opposite the motion Of the recirculatedair dampers ard outdoorait contol plan attempts fo mathe ‘the mixed-air tempera- ure ue point A at approximately 13 t0 14°C since We basic function of the air {0 tioning system {s to provide cooling. Another “equitement of the outdoor condo sure that the minimum perentage of ONE! A maintained. The SFopram to accomplish these several reqivene hewn in Fig. 5-4. At a high Progam raperature the dampers provide the minim tm rate of outdoor air. At aoe rae temperature Tower than about 24°C (or WAS the return-ir tempet- an omg) itis more economical in cooling enersy f° We 100 percent outdoor sis. For ao ie temperatures below 13°C the damper: oper ‘themselves to maintain utdont mpersture of 13°C. To hold a mixedai emPonAS ‘of 13°C at extremely {ou outdoor temperatures, the fraction of outdo ‘could drop below the minimum. sow ato are therefore designed to hold to that mink ‘and allow the mixed-air ‘outdoor air Dampers Recirclated ‘Exhaust at Figure $3 Ovtdoor-ir contol = AIR-CONDITIONING SYSTEMS 91 se Outdoors temperature, Figure $4. Outdoorair contol temperature to drop below 13°C. The paeumatic control system to achieve this plan is described in Sec. 9:10. Example $1 If the outdoor-ir controller is to maintain a mixed-air tempera ture of 13°C and a 20 percent minimum percentage of outdoor air when the fecirculated-air temperature is 24°C, at what outdoor temperature do the dampers Close to the minimum 20 percent position during cold weather? Solution An energy balance for the mixing process when the dampers have closed to the 20 percent position is, 0.204, + 0.80(24) .00(13) ‘The outdoor-air temperature fog is 31°C. ‘Since this outdoor-air temperature of -31°C is lower than experienced in all but the coldest locations, sometimes the provision to hold the minimum outdoor air rate at low outdoor temperatures isnot even incorporated into the outdoorir controller. “The shift to minimum outdoor air when the outdoorair temperature exceeds the recirculated. temperature is made to conserve energy. At outdoor-ir temperatures fbove this crossover point itis more economical to condition the recirculated air. An air property that is more decisive than temperature in predicting the refrigerating rate at the coil is the enthalpy. The psychrometric chart in Fig. 55 shows two triangular regions, X and Y, where the enthalpy-control concept makes jucements differing from the changeover controlled by comparative dry-bulb temperatures. When the outdoor air conditions le in region X, the temperature changeover would choose outdoor air, even though the outdoor air, because of its high humidity, requires greater cooling capacities than if recirculated air were used. Als, ifthe outdoor ar lies in region ¥, the temperature changeover uses reciculated air, even though the low-humidity out- {door air would require less cooling eneray. While the traditional practice has been to try to achieve a mixed-ar temperature Year round of approximately 13°C, current practice is influenced by the desire to 92 REFRIGERATION AND AIR CONDITIONING ‘cooling coil contion | 2 irene pS = condtion vig |” I Temperature igure $5 Using te compaon of ar enthalpies 10 sich between 100 pssst an main ‘outdoor ait conserve energy; 0 the mixedalr temperature may be rest to higher value if and conor cooling loads can be met by higher‘emperature ai 5:4 Singlezonesystem design calculations When « conditoned spase C™PeSt, 0 5-4 Singer both sensible and latent heat from interne and cxteros eas, the Mepply aif must enter the conditioned space with both s semPers ‘and humidity soppy ait ina the values to be maintained in the space. 1f qs the sense cooling rato ow watts and the latent lad, any entering pot f long the wd an line done tag. 5-6 wil provide the proper proportions of sensible and latent cooling provided that eet, Me Rhy at %, ey nthalpy, IDK {= temperature, °C ep = seciicheat of ar = 1.0 KOkg* K where Tamidity tio Figure $6 Load-rtio line for cooting 288 “Temperature ddumiying assienment. 5 [AIR-CONDITIONING SYSTEMS 93 Homiaiy ratio Figure 57 Ste points of ir during Cooling and dehumiiying with a singe- Temperature one ai conditions “The combination of the supply condition i and the flow rate of the air w must be such that the sensible-and latentheat loads are satisfied % 2% t4 we gle 62 ele“ heh, ‘the sensible-and latent-heat gains q, and a, are only part ofthe heat the cooling ‘coil must remove, since there is also the cooling load attributable to the ventilation Sir. This additional load is evident from the psychrometric chart in Fig. 5-7 because the assignment imposed on the coil is to cool and dehumidfy the mixed sir to @ point that lies on the load-ratio line * Example $2 The sensible- and latent-heat gains in a space served by asingle-zone sir conditioner are 65 and & kW, respectively. The space is to be maintained at 24°C and 50 percent relative humidity. The design conditions of outdoor fre 35°C dry-bulb and 25°C wet-bulb temperatures. For ventilation purposes ‘outdoor air is mixed with recirculated air in a 1:4 proportion. When mixed air t the resulting conditions enters the cooling coil, the outlet air conditions are a function of the temperature of the chilled water supplied to the col, as indicated in Table $-1, Determine (a) the air conditions entering tke coll, (6) the air condi ‘Table 5-1 Outlet sir conditions from cooling collin Example 5-2 ‘Als leaving temperatures, °C Chitedwatersupoty temperature, C ay bulb Wet bulb 40 107 105 30 ust 60 ns a 70 133 2 (94 REFRIGERATION AND AIR CONDITIONING Sw. anid ato Figure $8 Prychrometsic chart Bs Es ‘Sowing properties of alt in Ex: “Temperature, °C angle $2. tions leaving the coil and the required temperature of the supply cilled water, and (¢) the cooling capacity of the coil. ‘Solution (a) On the psychrometric chart in Fig, $8, four parts of return a seen T (AC and 50 percent relative humidity, A= 47.5 Kika, W = 00089 Tepes) mix with one part of outdoor air at point 2 (35°C dry bulb temperatire xa) bulb temperature, h = 76.0 Kifkg, W = 0.616 klk) From energy and 20 ts balances the enthalpy and humidity ratio at point 3 after mixing are iy = 0.8(47.5) + 0.2(76.0)= $3.2 K3/ks W, = 0.8(0.0093) + 0.2(0016) = 000106 ka/ke ‘atthe point located by these valves of and WS, oer properties can be deter mined: Day-bulb temperature = 26.2°C Wet-bulb temperature = 18.8°C (These ae the air condition entering the coi to whieh the performance dat in ‘Table 5-1 apply.) i 1) The loalsatio line extends downward and othe lft rom point 1 with 2 stops such thatthe proportions of sensble- and latentheat removal are sasted ' sore Sine load-satio line choose arbitrary a tempeaturef (Fg. 56) of 14°C and qhen compute a value off at that temperature. From Eq. (5-1) 36.3 Ki/ke Connecting the two points 3 and gives the loadzatio line which intersects the ‘ saturation line on Fig. 5-8 at 12°C. oa 3 AIR-CONDITIONING SYSTEMS. 95 ‘Table 5-1 indicates that a chilled-water temperature of 5.5°C results in outlet- air conditions of 12.1 and 12.0°C dry- and wet-ulb temperatures, respectively, which satisfies the load-ratio line. (©) The enthalpy of air leaving the coll at dry- and wet-bulb temperatures of 12.1 and 12.0°C, respectively, is 34.2 Ki/kg. The mass rate of flow of supply sir to the space is, from Eq. (5-2), 65+8kW ATS WaT ‘The enthalpy of air entering the cooling coll is $3.2 Ki/kg, so the cooling capacity required of this col is (5.49 kes) ($3.2 ~ 34.2 KI kg) = 104.3 KW ‘The difference between 104.3 and the room load of 73 KW is attributable to the cooling load of the outdoor ventilation air. 5.49 kg/s 55 Multiplezone systems For large buildings itis usually not economically feasible to provide a separate system for each zone. For such cases the basic eentral-system concept is expanded to meet the cooling and heating requirements of multiple zones. ‘A zone may be a single room, one floor of a building, one side of a building, or the interior space. Essentially a zone is the space controlled by one thermostat "A wide variety of combinations of duct networks, coil locations, and control strategies are in use, but the most common are: $ 1. Constant-volume systems @ Terminal-reheat . Dual duct or multizone 2. Variable-volume systems 4, Single-purpose cooling or hesting 2. Cooling with reheat ¢, Dualduct variable-volume $4 Terminalreheat system The schematic diagram of the temminal-eheat system is shown in Fig. 5.9. All the air is cooled to a temperature of perhaps 13°C to assure ‘dchumidification, and the thermostat in each zone controls the reheat coil associated ‘with that zone s0 that the temperature of the entering air will be such that the zone temperature is maintained. The reheat coil may be hot water or electric. The advantages of the terminal-eheat system include the small space occupied by duets and excellent temperature and humidity control over a wide range of zone loads. The primary dis- advantage is the relatively high energy requirements for both cooling and heating. ‘The energy penalties may be partially overcome by raising the temperature of cool supply air until one of the reheat coils is completely off. Ancther means of reducing the energy penalty is to perform the heating with recovered exergy! from some other part of the system such as the reftigerant condenser or lighting, 196. REFRIGERATION AND AIR CONDITIONING Figure 5:9 A terminabrcheat system 46:7 Duslduct or multizone system In the dusldut system, Show schematically in fig, 510, the air from the supplysir fan divides, Part of air flows through the ae ing call and part through the cooling col. The thermostat ‘each zone regulates heating eepox that proportions the low rate of warm and cook 2 ‘maintain the a. mini Dypeatre in the zone. The dual-doctaysem i vey sespong © changes in desea tne zone and can simultaneously accommodate eating 2 1 ‘ones and reat in others. One disadvantage of the system is the spent of two supply-air seeteE oth of which must be large enough to handle a ihe airflow. As with the ducts, bovheat system, there wil be periods of simultarcous AeatOe ‘and cooling Lerma se ge energy elficiency of the dualduct system. 09 Oe other hand, wach re outdoorairtemperatore is low enough to achlee the 13°C without operat The the cooing col, some energy can be conserved. Good Soe effectiveness is also ing the efarng hot weather i the temperature ofthe wann duct I set ow or-even petter-the heating coil is shut down. cane tizone system is thermally Wendel to the dualduct Se but the confguation differs in tht all the mixing boxes are chstered af the central unit cor individual ducts convey the mixed alr to each zone, Figure $10 Duabvet syste. [AIR-CONDITIONING SYSTEMS 97 Example 5-3 A zone served by the dual-duct system in Fig. 5-10 has a design heating load of 8 kW and a design sensible-cooling load of 6 kW. The zone is to ‘be maintained at 24°C, the temperature of the alr inthe cool air duct is 13°C, ‘and the temperature in the warm air duct is 40°C. Assume that the temperature ‘of mixed air is 24°C. Ata partload condition when the sensible cooling load is, ‘3KW, what are the heating- and cooling-energy rates attributable to this zone? ‘Solution It is fist necessary to determine the airflow rale to the zone, a value that in the dual-duct system remains constant forall load conditions. To meet the design heating load the required flow rate is (8 kW)/[(40 ~ 24°C) (1.0 KI /kg, 'K)} “0.5 kals, and to meet the design cooling load (6 kW)/{24 ~ 13) (1)] 0.55 kg/s. The flow rate needed for the design cooling locd controls, so the ai flow rate is set at 0.55 kel. ‘When the sensible-cooling load is 3 kW, the air temperature entering the zone is 24°C ~ (3 kW)/[0.55(1.0)} = 18.55°C. The energy balance of the ait- stream in and out ofthe mixing box is w_(03°C) (1.0) +, (40) (1.0) = 0.55(18.55) (1.0) where w, flow rate of coo! air kel 1g = flow rate of warm ai, kls Since wy, = 0.55: Wes aw, + 40(0.55) - 40w, = 10.20 “Then w, = 0.437 kgs and w, = 0.113 kg/s. The energy rete needed to bring the (0.113 kes up to 40°C in the heating coil is 0.113(40 ~ 24) = 1.80 kW, and the energy rate in the cooling coll is 0.437(24 - 13)= 4.80 KW. ‘Because of “thermal bucking” of the two coils, the cooling coil must extract not only the 3-kW cooling load of the zone but also the 1.8 KW introduced by the heating col 5.8 Variableairvolume systems The poor energy characteristic, especially during light heating or cooling loads, of the constant-volume air systems discussed in Secs. 5-6 and 5.7 have shifted preferences in new designs to variable-air-volume (VAV) systems, ‘There are a number of variations of VAV systems and also a number of possible com: binations of VAV with other systems. Three important configurations are (1) cooling ‘or heating only, (2) VAV reheat, and (3) VAV dual duct in the cooling-only system, as shown-in Fig. 5-11, a single steam of cool air serves all the zones, and a thermostat in each zone regulates a damper to control the flow rate of cool airinto the zone. The desirable energy characteristic of this system is that at low cooling loads the flow rate of air is reduced so that the required cooling capacity atthe eal is correspondingly reduced. The cooling-only VAV system is widely lused for interior spaces of buildings with no heating loads and where only cooling loads prevail. The system experiences a problem at very light cooling loads where the airflow rate drops off so much that poor air distribution and/or ventilation results. ‘The heating-only VAV system has the same structure as that in Fig. 5-11, but instead of a cooling coil a heating coil provides a source of constant-temperature 98 REFRIGERATION AND AIR CONDITIONING igure 5-11 A coolingonly VAV system. ‘warm air, The conditions adaptable to s heatingonly VAV system are rather raze in ‘building air-conditioning systems. Sn veheat system is identical othe one shown in Fig. S-11 except that he ran ine to each zone contain a reheat eo. The contol sequence i that as ving load dzops off, the damper progesively reduces the flow rate of vat soe to 30 percent of full low rate. At this point the aisflow rate remains C07 snot and the reheat col is activated. There is thus some thermal buckiog 0 ®t stant Snventional termina-ceheat system, but it occur at 2 reduoed aisow 1 and oe enuits in only 2 modest loss of efficiency. The VAV reheat system overcomes t ts es cefenles of te coolingonly VAV system since it provides a means 9F aotaining adequate ar distbution and ventilation without paying the ener penalty fncurred in constant-volume reheat applications. re SAY Guat system, the arrangement is sma to the convention) uaiuct system of Fig. 5-10, except for the low characte of the mixing Po1S* dae iding a constant fw rate of mixed ai, te dampers are ranged £0 se racm and coo airflow rates drop appreciably before the other streim Soktt Thown in Fig 5-12, is that the airflow rate to the zone is to supply air. The result, 28 ‘of control characteristics the desired minimum airflow variable, but by proper choice Airflow ate “hiiow rte Ful Ww Fut heating coating igure $12 Aisflow rates in a VAV Space temperature,“ 2s duakeuct system, [AIR-CONDITIONING SYSTEMS 99 rate is asured. The characteristics of modulating controls, discussed in Chap. 9, lend themselves to achieving the flow rates shown in Fig. 5-12 by petting a span of space tir temperature. In Fig 512, for example, the VAV mixing box provides the neces: sary flow rate of warm air for full heating when the space temperature is 21°C, When. the space temperature is 25°C, the cool-ar dampers open enovgh to meet fll cooling load. ‘The VAV cheat and dual-duct systems provide all the flexibility of the con? ventional reheat and dua-duct systems in the sense that a zone can be accommodated rit switches from heating to cooling requirements and that some zones ona system an be provided with heating at the same time that other zone: need cooling. There is Soin thermal bucking in both the VAV reheat and dual-duct ystems, but the magni- tude of heating- and cooling-energy cancelation is modest becuse of the low afflow rates at which the thermal bucking occur [Example 5-4 In a certain VAV dualduct system a flow sate of warm ait of 0.8 Jigis is required at fll heating load when the space temperature is 21°C; the space requires 1.1 kis of cool air at full cooling load, which is called for at a space tempersture of 25°C. It will be convenient to choose control equipment tht has the tame slope of the airlow-to-pace-temperature lines for both the warm and ool ar (ne slope is the negative ofthe other. If the minimum seflow rate i to te 03 kg/s at what temperatures do the warm airflow rate and the coo! airflow rate fall to 2210? Solution Let m= slope of the coolsirflowrate line, asin Fig. 512, and -m the slope ofthe warmairflow-rte line. Then wee, tit, and wy =e, -m, where We, wy = airflow rates of cool and warm sr, respectively, kas gest = const ‘= space temperature, °C ‘The 100 pereent cooling and 100 percent heating conditions provide the expres: 116, #25) 3) and 08 =c, -m(21) 4) ‘The minimum siflow rate will occur when either w, or wy has dropped to zer0; thus. - w,20= 6,4 mio 65) where foi the temperature where w, 0, at which condition 13 264 Mp 66) Equations (5-3) to ($6) ae four simultaneous equations that can be solved for Ww, = minimum airflow 100 REFRIGERATION AND AIK CONDITIONING the four unknowns Ge ¢y» and fp. The esl are eqe-89 G792 ma0d and fyg"2225°C sme flow rte of warm air drops t 210 when fi to 23°C, 0 between 22.25 ‘and 23°C the mixing of warm and ‘cool air results in the minimum total airflow rate of 0. kgis. ems accomplish heating and cooling through the disti- Fos of water alone, although the final heat transfer inthe conditioned space mist ution or rom air, Outdoor al: for ventilation must be provided and conditioned it ee or aco uit, unit ventilators, or convectors ar the most common termina! cae ae by water piping systems, Water systems occupy relatively little space sod uni nthe lowest Rustcost systems avalabe. The systems usually lake unity ae vl and ventilation may be uncertain even if outside openings are provides 1 corte ul unit, Wind pressure, the stack eect in tll bung, and the possibility ca ring the cols in cold weather all equie special precautions when ovtdoor-¢ er cungs ave provided Since condensate drains must be provide at each col Tot Oren ape a more significant factor than in air systems, where the dehumiifics tion can be accomplished at a central location. cae Tuite can be served by two- or fourcpipe watz-dstibution systems. 6 worrpe astem serves units with a single cil andthe system can provi bet br reer: putit is not posible to heat some zones while simultaneously Coots others nee bipe system (ig. 513) serves fan-coll units with «wo cals, one for 5.9 Water systems Water sys Presureselif [rare Fourie fan rans Return CChiled-water Feta outdoor sit Blower pe water thermaldstibution system. Figure $:13 The fou >? we AIR-CONDITIONING SYSTEMS 101 heating and one for cooling. The hot- and cold-water loops have their own supply and return pipes. The space thermostat regulates the flow of hot and cold water to the Coils, but the control is sequenced so that the hot water is off before cold water is ‘admitted to its coil, and vice versa. Water systems may also serve convectors that have no fans (see Sec. 7-4). Con- ‘vectors are used almost universally for heating and rarely for cooling, because of the problem of draining the condensate. A popular application of convector sin pevim- ter zones of buildings that are also served by a VAV system. In this arrangement the VAV system can be of the coolingonly type with all the heating provided by the convectors 5-10 Unitary systems The systems described to this point have all incorporated Contrally located, field-assembled, heating and cooling equipment. Multiple-unit or itary systems, on the other hand, are factory-assembled unit located in or near the conditioned space. These systems are available as a single package containing direct expansion evaporating coils, controls, fan, compressor, and condenser or they may be split units with the compressor and condenser located remotely. ‘When appropriately applied, multiple-unit systems offer a number of advantages. ‘The fact that they are mass-produced and factory-assembled usually means lower first cost and lower installation costs, With proper selection and control they may also provide relatively low operating costs, By their nature multiple-anit systems are zoned Systems. These units are manufactured with matched components and usualy with certified ratings and published performance dats ‘The disadvantages of unitary systems are that there are relatively few options with respect to sizing the evaporator, condenser, fans, compressor, and controls. Since tach unit must be capable of meeting the peak load ofthe space it serves, the installed ‘capacity and connected electrical load are usualy large than with a central system. ‘Examples of the unitary air conditioner are window units, through-the-wall units, rooftop units, and split systems. Window units are primarily found in residential ‘pplications, and they have no ducted air distbution. The appearance and noise of ‘these units Limit their application, Through-the-wall units are generally more acceptable than window units from the standpoint of appearance since they are designed into the building. They find application in motels, healthcare facilities, schools, and sometimes offices, Rooftop units (as shown in Fig. 1-2) are primarily applied in low-rise buildings ‘with fat roofs, such as stores, shopping centers, and factories. For better air distribu: tion in the conditioned space the conditioned air should be ducted from the rooftop ‘unit to multiple outlets instead of introducing the total airflow rate at one postion “The split system, serving such installations as small stores or an office suite, usally conveys the conditioned air through ducts to the spaces served by the conditioner, PROBLEMS ‘51 A conditioned space that is maintained at 25°C and 50 percent relative humidity experiences @ sensible heat gain of 80 kW and a latentsheat gin of 34 kW. At what temperature does the load-ratio ine intersect the saturation line? Ans, 9°C 102 REFRIGERATION AND AIR CONDITIONING 52 A-conditioned space receives warm, humidifid air daring winter a comsioniss ~ Soa arasncinain 20°C and 30 percent relative hum. The space expense 95 = in ordetign rate of 02 kel of outdoor as and an additonal snsiic ne Ss of 25 infiltration Moor airs saturated a a femperature of 20°C (se Table 7) If con- XW ris suppied at 40°C dry-bulb temperature, what must the wet-on {emP=r Sint supply air bein order to maintain the space condtions? Ans: 18 °C sere iaporstory space to be maintained at 24°C and SO percent reais MoS 53 Monona a aenablecooingJond of 42 KW and ata oud of 18 00 Dost the experienc sc peay, the airconditioning system is eqipped for repeating 1 2 latent 10 SSoling Coil The cooling col has been seleted to provide ous 8 ea ang percent relative humidity. What i (othe temperature of ss7PIy ‘rand Gp) the airflow rate? Ans. (0) 3.8 ka/s ne puning outdoorair control Sec. §- explained that with outdoosconsitions aoa eid Y regions on the pryehrometric chat in Fig 5-5 enthaly control To/* in the rjent We now explore some Linitations ofthat statement wilh Tres © neray files pone that te temperature sting of elt ar fom the cooing ee ected that the outlet air is essentially saturated when dehumiciicato“ Sot 5 10°C ae te contin of glum ars 24°C and 40 percent relative Mami) A inn jor conditions are 26°C and 30 percent relatie humidity, would reen 2 ar oatside ait be the preferred choice? Explain why. or oat inal reheat system (Fig. 59) has alow rats of sappy tr of 18 Aas St SA termina Trung with 3 els of ouside air at 28°C and 30 percent rob currently 6 Pepin sensible Toad inthe sacts is 40 kW, andthe ens 8 aay npne temperature ofthe supply ais constant at 13°C. An accuse) of the aoe og te building wap shocked by the uity bil and Onder 27 firm occuring om 24 to 25°C, What i) the ateof het removal in hs cPooTe thermostat tage the change and (8) the rate of heat supplied a he hee Cot coll before ae the change! Assume thatthe space seb Toad remains conse #1 befor ame (a) 15 2W increas in cooing rates (0) 18 XW incease in esting aie REFERENCES 1.1.6. Oleg Enegy Rechining Modular SleContaned wagon Unt ec Con. 7 Te VAC Equip, Components, Pardue Unir October 78.1974, Pp go) 02 aoe ete ie erent, ASHRAE Syn ASHRAB Trot, vo 80, BPP 47205, 1974. 1 or sytem to Cope onthe Inherent Brey Conening Fetes of VAY SHEN 5 can AE Symp ASHRAE Trans, vl 83, t. 1, pp. $8111, 1977 1 DSEERAE So ir Volme Terminal, ASHRAE SY. ASHRAE Trans, vo 86 2 pp. 825-858, 1980 Foeaaaa seer oduct Decor, Systems Volume: Amadcan Society of Heating Rete: aeagd Air Conditioning Engineers, Atanta, Ga 1986, SIX FAN AND DUCT SYSTEMS 61 Conveying air Chapter 5 explained the arrangement of the popular air systems (Gariable-olume, terminaleheat, ec.) and thi chapter follows up by concentrating tn four topics associated with the flow of ai in an air system: (1) computing pressure {drops of air flowing through ducts and fitings, (2) extending the computation of pres Sure drops to designing a duct system, (3) understanding the characteristics ofa fan, both independently and in conjunction with a duct system, and (4) designing the distribution of air ina conditioned space. ‘Since the fan motor is a large consumer of energy and the duct system occupies considerable space in the building, the fan-duct system deserves keen attention during ‘esign, Unfortunately, because there are dozens (or hundreds) of interrelated decisions in the design of a fanand-duct system, most designers are satbfied with achieving 2 workable system and do not proceed further toward a combined optimization of life lime energy cost, ductaystem cost, and building-space cost of the fan and ducts Computeraided design (CAD) should bring improvements in the future, but even CAD must apply. principles correctly; some of these principles will be explained in this chapter. 62 Pressure drop in straight ducts The fundamental equation for computing the pres: sure drop of a fluid flowing through a straight duct of circular cross section is 1) where Ap = pressure drop, Pa {f= friction factor, dimensionless L= length, m 103 104 REFRIGERATION AND AIR CONDITIONING ‘D= inside diameter (ID) of duct, m = velocity, m/s - p= density of uid, kg/m? st ison factor fsa function ofthe Reynolds number and the ative 008s ess oh pipe surface e[D, where ci the absolute rousness in meters. Both gap a ron zepresentations of te fiction facto are avaale, Equation (67) derives from the work of Colebrook,* - 2 1 ph re 62) 2ilog f+ a) °F |! * Reet) VF ss equation ipl inf stv of can be subsatban he it dis Te caiaoved valve computed from the equation. Tee definition ofthe Reyaolis number is jaar ae? vo, Rea 63) # _where isthe viscosity in pascal-seconds. Tet ea source ofthe value of fi the tational Moody chart! reproduced sn Fig. Gh Th absolute roughnese of some surfaces shown in Table Example 641 Compute the presure drop in 15 m of stnightcrslar shes smetal Exams am in diameter when the flow rte of 20°C airs 05 m/s Solution The velocity Vis ya 0smh nOs7V4 = 7.01 m/s ‘Table 6-1 Absolute roughness of ‘some surfaces some surfaces Material ‘Roughness em Rivete steel £0.0009-0.009 Concrete 0.0003-0.003 Cast fon 0.00025 ‘Sheet metal 00015 Commercial steel 0.000046 Daawa tubing 8.000001 OO Figure 6-1 Moody char for determining Histon fer aque mousey 100'000 ua Et “oT HUIS ITE wood 95> EGONZ OLAS y ECO OL 500 600 to so‘o00 000 sto 000" raid woous 3000: 8000 Et co z 20 se t no eo & 900 le 00 0 7 sto wo s0 0 7 0 io so E saiduino! susqnr aGuo9: itfimeuy9: 28 ¥ Yo jnoy UT Eo sou 10s sopes wonstsg 106 REFRIGERATION AND ALR CONDITIONING ‘Table 6-2 Viscosity and density of dry air ‘at standard atmospheric pressure ~ Temperature, Visosity a, Density, °c wars alm? -10 16.768 13414 : ° 1238 (12922 10 17708 (1.2467 20 1ei7s 1.2041 30 teese 11644 0 igaleLia72 50 igses 10924 2 ee “The density and viscosity of dry air at standard atmospheric pressure are presented in Table 62, At 20°C the viscosity His 18.178uPa +s and the density p is 1.2041 ikglm?. The Reynolds number Re is 07 m/s) (03 m) (1.2041 keln® __ (707 mls) 03 m9) (1.2041 KB) «49 599 18.178 uPa > s ‘The roughness of sheet meta, from Table 6-1, 0.09015 m;so the relative rough hess eJD is 0.0005. From the Moody chart, Fig. 6-1, the friction factor is 0.0195, ‘and the same value results when using Eq. (6-2). ‘The pressure drop Ap in the 15-m length is 157107? 032 Re ‘4p =0.0195 (1.2081)= 29.3 Pa “To facilitate computation of the pressure drop in «duct, graph similar to Fi. 6-2 a1¢ available. ‘63 Pressure drop in rectangular ducts Because rectangular ducts (Fig. 6-3) are 50 Sraely used in air-conditioning practice, an equation for the pressure drop in ree- angular duct is necessary. A convenient form of the equation is paaar0d Le . apt Pe oH. m2 where Daq isthe equivalent diameter ofthe rectangular duct in meters, AN eAPreSO8 | for Dy, canbe developed by observing that for the cular duct te diameters 4X cross-sectional area _4(@07/4) perimeter » ‘Using the same expression for the rectangular duct yielis X crosssectional area ab __2ab fe perimeter Ward) a+b é Rs 35,4014 Vs ON? Lo Lo og SAB ero ot ial ates orb aet 307 8 os I os os % os oa ht 7 os = 03] oF . 2 03 3 = og] on 2 aas| jos ong a0 oe oe 0.04 006 005 os 4 loos oo os rr) oo: ons ons oo oon 00 Rist c.00s 0.005 IZ APS orcad thifoone ar 02” oaosoar 152 34 6 8101520 30 $0 100 Friction Loss, Palm Figure 6-2 Presure drop in straight, circular, sheet metal ducts, 20°C air, sbsolte oghness 0.00015 m 107 108 REFRIGERATION AND AIR CONDITIONING a jt aA i ron Home oin se ‘The equivalent diameter calculated by Eq. (6-5) can be used in conjunction with Fig. 62 in'a special way. Figure 62 canbe used ifthe chasis enteed with thea) vanity and the equivalent diameter calculated from Eq, (65) The airflow rate, how- ‘ever, applies to circular and not rectangular ducts. tee able to use Fig. 6-2 for rectangular ducts when entering the chart with the ftow sate, a new equivalent diameter Deg, must be detemined. To find Dear fst some the pressuredrop equation in terms of the flow rate Q m/s instead of the Woety. Using. an expression for the fieton factor of /= C/RE®-?, we get for 2 cireular duct ( 2 cL \ sD, TRO 66) D%y and for the rectangular duct, where V= Qab, ap: c L AP" Tr 3ap\ OF Dab 2 Cy a+b atd abe ‘The pressure drop in the rectangular duct calculated by Ba. (6-7) wil be he sme 38 dt For Eq, (6-5) and Fig. 6-2if the following relation forthe dimensions holds 1 6 (tyra 1 Grp \ 2) at OP Thus (a9 Dag 7 130 Saas 68) 2a. 130 4 F 68) With Dag Fig, 62 can be used to caste the pesue drop by entering he chart dnecdtflth the flow rate, With this procedure, che velocity indzated by the ‘chart is not applicable but can be calculated from ‘QIA. Equation (6-8) holds? for ‘vidth-to-height ratios up to about 8:1 FAN AND DUCT SYSTEMS. 109 Example 62 An airflow rate of 1.5 m/s passes through a rectangular duct 0.3, by 0.5 m, Calculate the pressure drop in 40 m of straight duct using (2) Dag and ODay Solution (a) D, ab 03405 From Fig. 6.2 with a velocity of 10 m/s and an equivalent diameter of 0.375 m ‘the pressure drop is 3.0 Pajm, Note that the airflow rate of 1.2 m3/s indicated by the chart is not applicable In 40 m the pressure drop is 120Pa. o oes 20 C20 OT rs “042m From Fig. 62 with an airflow rate of 1.5 m3/s and an equivalent diameter of, (0.42 m the pressure drop is again 3.0 Pa/m, Note that the velocity indicated on the chart of 10.8 m/s isnot applicable. The correct velocity is 10 mis (64 Pressure drop in fittings An airchandling system consists 0° straight duct and fit tings. In the fittings the air undergoes changes in areas and direction. These fittings include enlargements, contractions, elbows, branches, dampers, filters, and register, ‘The al-pressure drop in these fittings must be known in order to design a system properly. In actual system design, pressure drop in these fitings may be of more con ‘erm than that in the straight duct that connects them. For example, pressure drop in an elbow may be the equivalent of 3 to 12 m of straight duct and could be as high as 20 m. Care in estimating the pressure drop in fittings is therefore justified, although unfortunately the type and quality of construction have a pronounced influence on the pressure drop of a given fitting. Since fittings occupy only short lengths along the flow network~generally less ‘than I m-explanation of the pressure drop cannot be attributed to the drag along the surfaces of the duct. It is due instead to momentum exchanges between portions of the fluid moving at different velocities. More specifically, at scme postion in the fit- ting the fluid experiences a sudden expansion, and this process will be used to explain, atleast qualitatively, some of the trends in pressure-drop characteristics. Changes in area and direction that will be explored in this chapter are sudden expansions and contractions, elbows or turns, and branches. 65 The V2p/2 tem A grouping that has already appeared in Eq (6-1) will resin the calculation of presure drops. The pattern that wll emerges thatthe pressure loss for an incompressible Tud is the product ofthe ¥2p/2 group and aterm that char- 110 REFRIGERATION AND AIR CONDITIONING eee wee Figure 64 Flow through a conversing uct setion. ‘cterizes the geometry of the duct or fitting, Thus, from Eq. Lo When air lows fritionlessly through a converging or diverging nozzle, as in Fig, 64, the Bernoulli equation (2-8) applies 4 28 2 es) Since ree v20{/4,¥ rey ‘ay a ) wl? Geoistey “The next several sections will show thatthe ¥2p/2 group also appears in expresions pee pe eeure drops which represent loses, in contast 10 Eq. (2-8), where theres tho pressure los, only pressure conversion. 64 Sudden elargersent What i almost a building block for predicting the presse te Seihtings isthe relation desrbing the pressure Jos ina sudden enlargement 35 se ne gs. These Is change in are, asin the corverging section of Fig 4, eae avez of loses that prevail the Bemouli equation no Tonger applies. can be mended tothe revised Bernoulli equation V4 P11 0P2 4 2 Pow 10) pia ep 2 e -the other principle to be combined with Eq (6-10) to develop an expression forthe | ks tt a0 dr>0009 , FAN AND DUCT SYSTEMS 111 pressure loss isthe momentum equation Py ~PyAy = Ve(¥a42~—)* A?) ou) which states thatthe difference in force on opposite sides ofthe control volume shown by the dashed lines in Fig. 65 equals the rate of change of momentum. The not 30 ‘bvious term in Eq. (61) is py. Due to separation ofthe flow from the surface at fhe abrupt enlargement the low pressure p prevals immediately after the expansion’ ae sets over the entire area A>. It is this separation that causes the pressure loss in ihe sudden enlargement. Substituting py ~ P2 from Eq, (611) iato Eq, (6-10) yields the expression for Pos, ef AY =? (3) pe 5-12) Pros => (: +) 12) ‘The pattern appearing in Eq. (6-12) is now familiar in that the loss is the product of the V2p/2 group anda term representing the geometry. Equation (6-12), called the Borda-Carnot equation, agrees sufficiently well with experimental results to be used for ductsystem design. At high airflow rates the equation gives losses that are several percent high* and at low airflow rates, several percent low. Example 63 Air at standard atmospheric pressure and a temperature of 20°C flowing with a velocity of 12 m/s enters a sudden enlargement where the duct area doubles, What is the increase in static pressure of the air asit passes through the enlargement? ‘Solution From Eq. (6-12) the pressure loss due to the sudden enlargement is (42 m/s)? (1.204 kgim?) yee 2m ati (ip 17a Substituting Pig into the revised Bernoulli equation permits computation of the pressure rise Wi-VDe 12-6? Py -Py Pr 2 2 433Pa Instead of the pressure rising 65 Pa, as the Bernoulli equation would indicate for & process with no losses, the actual pressure rise i 43.3 Pa, (1.208) = 21.7 16-7 Sudden contraction A sudden contraction occurs in a duct section where the duct fize is abruptly reduced in the direction of flow. The flow pattern in a sudden contrac- tion, as shown in Fig, 6-6, consists of a separation of the fluid from the wall upon ‘entering the reduced cross-sectional area, and a vena contracta forms at 1'. The con- cept in predicting the pressure loss is to propose no loss from positions 1 to 1’ and to ‘treat the flow from positions I’ to 2 as sudden enlargement. This loge is quite valid, 112 REFRIGERATION AND AIR CONDITIONING ff Fiue6s Asiddenconincion —* since converging, accelerating flow is efficient, while deceleration of fluid is dificult to achieve without losses. Borrowing from Eq. (6-12), then, we find the pressure loss in the contraction to be ware at) Plog #1 13) ws 2 A, ‘The area of the vena contracta can be related to Az by defining a contraction co efficient C. ott 618) oa, iy Cy Substituting Ay’ and ¥'from Eq, (614) into Ba. (6-13) yes Ve fi_\? Pros ze D) 15) "The contraction coefficient isa function of the ratio ofthe areas, A/A it was deter- ‘ined experimentally by Weisbach* in 1855 and is shown in Table 6-3 ‘The form of Eq. (6-15) again shows the pressure drop to be calculated by the Vop!2 group multiplied by a geometry factor. The geometry factor reaches a maxi- mum of approximately 4, which may be compared with the maximum value of the ‘Table 6-3 Contraction coefficients in sudden contractions An Ay & 01064 020832 03 O6as 04 0459 0s osst os ona 07 07s : os 0813 09 (0892 101.000 FAN AND DUCT SYSTEMS 113, ! = PR Leal ! Fk Figure 6-7 Separated flow inan elbow. geometry factor for a sudden enlargement of 1.0, In any sudden change in area for & fen rate of flow there will be a greater pressure loss ifthe Mui flows from the small fo the large area than when it flows in the opposite direction. 68 Tums The most common elbows used in duct systems are 90° tums, either ciicular of rectangular in cross section. Weisbach* proposed that the pressure loss in an ‘tbow is due to the sudden expansion from the contracted region in plane 1’ in Fi. 6-7 to the full cross-sectional area of the duct in plane 2. If this proposal is correct it should be possible to express the pressure los in terms of the V2p/2 group. The mag- nitude of the Reynolds number has an influence but not a daminant one. Pressure losses in rectangular elbows determined by Nadison and Parker” a shown in Fig, 68. These data also show that a flat 90° elbow with a large value of WIHT suffers less pressure drop than a deep 90° elbow, suggesting that the subdivision of an elbow into multiple elbows of large W/H by installing turing vanes will reduce 0 21 02 03 oF as Ratio of inner to outer radius Figure 6-8 Pressure losin rectangular elbows 114 REFRIGERATION AND AIR CONDITIONING liz. - the pressure drop. Turning vanes, as in Fig. 69, have the effect of increasing WIH and are frequently used, ‘The pressure loss in elbows of circular cross section” is summarized in Table 6-4 69 Branch takeoffs When a main duct supplies air to several branch ducts, a takeoft ‘must be provided for each branch, as in Fig, 6-10. From the upstream postion u there is pressure loss both tothe downstream postion d and into the branch to point b ‘Considering frst the pressure loss from u to d in the straight section of duct, ‘this loss occurs because the pressure buildup from the higher velocity at u to the ower velocity at d is less than the ideal. The pressure losin the straight-through sec tion of a branch takeoff is usually small compared with other losses in the system. In any low-velocity designs itis neglected, but an equation® that closely approximates tabular data i 4 Pon” HP 04) ( ny ra 16 ‘The pressure los“! fiom w to b expressed in term af Vo for sever afferent Ther of ako is shown in Fig. 61 ‘Table 6-4 Geometry factor in equation for pressure loss in circular 90° elbows (geometry factor) Pt ati absot emai " diameter Geometry factor Oooo Miered 130 05 030 073 04s, 10 033 1s 028 20 09 9 {Measured tothe duct centerline. yyd99 FAN AND DUCT SYSTEMS. 115 | i I | = = Figute 6-10 Branch takeo. Example 64 A 60°, 30- by 30-cm branch takeoff leaves «30: by S0-cm trunk ‘tact The size of the downstream section is also 30 by 50 cr. The upstream flow ane is 1.5 m/s, and the branch flow rate is 0 m/s. The upstream pressure is ‘$00 Pa and the air temperature is 15°C. (2) What is the pressure following the ‘Sraight-through section, and (b) what i the pressure in the branch ine? ‘Solution Velocities V,=10m)s ¥j2657 m/s Vy"556mis 971225 elm @ From Ea. (6-16) 6.677.225) (04) Pos ‘Substituting into the revised Bernoulli equation (6-10) gives 2 eyre(he 8 Pas) \p 22 5 10%(1.225)_6.679(1.228) = 5004 20229) SOT SE -12=533 Pa 4 Fi 7 2 8 é 5 4 3| g 4 1 os os os! 0. 7 34s 78S Oy040s05 08 | 7 Ratio of branch to upstream velocity ‘Fgure6-11 Pressure loss from the upstream positon tothe branch dct. 118 REFRIGERATION AND AIR CONDITIONING bya Figure 6-13 Mulibranch duet system, yr Suppose that the calculation results inthe pressure drops shown in Table 6-5. fan sopretpe selected to develop 92 Pa at the total flow rate the balancing damper left pen in tun 4-CG-H and the dampers in the other runs partially closed to provide 92 a pressure drop inal these runs atthe desired flow rates “An improved design result if one or more sections of run A-C-G-H are enlarged to reduce the critieal pressure drop. Its also possible to reduce the size of sections in the Sher suns while staying within velocity constraints, since excess pressure drop is available. paareaee 4613 Equalfeiction method There are several versions ofthe equal-fiction method, tat one that often results in a superior design is to select the pressure drop to be vu jable in the duct system and size the ducts to dissipate this pressure. The steps in this design method are as follows: 1, Decide what pressure drop will be availabe. 2: Compute the equivalent length ofall runs (he sum of the length of straight duct plus the equivalent length of fittings). 4, Divide the available pressure drop by the equivalent length of the run having the longest equivalent length. ‘4, Wish the pressure gradient from step 3 and the flow rate in each section of the Tongest run, select the duct size of all those sections, using Fig, 62 5, For the remaining sections, select the size to use the available pressure drop but stay within velocities appropriate for noise restrictions. pasaneas ‘Table 65 Pressure drops in the system in Fig. 6-13 Run Pressure drop, Pa AB 8 Acar 38 AcDF “a ACGH 92 acl 80 seer coecedee FAN AND DUCT SYSTEMS 119 In step 2 the equivalent length can be computed by dividing the coefficient ofthe Vap/2 tem in the equation forthe pressure loss of the fttrg by J/D for duct of the ‘hme size, Elbows might be equivalent to 3 to 12 m and tranch takeoffs 20 m of Straight duct. In step 5 some sections of the main duet might have been selected in Stang the sections in the ertcal run. If che extcal run in tie aystem shown in Fig. Bibs ACG, for example, the size of section A will be specified in step 4, so the Gressure available to section B should be computed and the size of B chosen to dis pate the available pressure ‘The equal-fiietion method usually results in better design than the velocity method because most of the available pressure is dissipated in friction in the ducts and [tings rather than in balancing dampers. The size and cos: of the duct are conse- quently reduced. 6.14 Optimization of duct systems The principal contributes to the owning cost of duet system are the costs of the duct and installation, insulation, sound attenuation, Srermy to drive the fan, and space requirements. The objective of an optimization Srecedure is to minimize total owning cost, detalled opimization study may be Tificult to perform, and in the design of small duct systems it may require more in ngineering cost than it saves in owning cost. On the other hand, in large systems an Gpuimization study may be a good investment, particulary ifthe value ofthe building space is considered. “x simple example of an optimization procedure would be to select the duct diam ‘eter that rrinimizes the initial plus operating cost of a duct system consisting of a fan sega length of straight duct, Total cost of the system isthe sum ofthe frst cost and the operating cost ‘Total cost = C= first cost + lifetime operating cost Estimators often use the mass of metal in the duct system asa guide to the cost ofthe fuct system, taking into consideration that the cost of insulation might be about ¢ dees the actual cost of the metal. For circular duct, initial cost could be represented by the expression Initial cost = (thickness) (nD) (L) (density of metal) (installed cost/ks) (6-20) where D = diameter of the duct, m L= length of the duct, m Fora given thickness of material, the expression for the intial cost ean be simplified to Initial cost = CDE 2 where C, is the constant combining the constant terms of Ea. (6-20). ‘Operating cost of the duct system isthe energy cost 2er hour multiplied by the number of hours of operation expected during the amortization period. The electtie power required can be calculated by dividing the air power by the efficiencies of the fan and motor, Since air power is the product of the Volume rate of flow and the pres 122. REFRIGERATION AND AIR CONDITIONING 1280 1 1OKW|15kW 2oKW 25K Ney iN 05 4 mt Spe \\ N \ 20 | \ 750) Presur incest, Pa s00| 250] ° 0 05 16 15 20 2s Flow sate, m/s a forwardcurvedblade contsifusl fan. The wheel igre 6-15 Perfoumance characteristics of Jas dmenions of the out are 0.517 by 0.289 m. Thmcter and the width are both 270 m. tion of the two power quantities is y poveiga "00,00 eas) “The fan efficiency is defined as the ratio of the ideal to actual power OWT jet Efficiency n= POWT cul e 65 Compute the efficiency ofthe fan whote characteristics are shown, Examp! "6-15 when it operates at 20 1/s and delivers 15 m°/s. in Fig Solution When the rotative speed is 20 r/s and the flow rate is 15 m/s, the fan Sour te the pressure ofthe ait S00 Pa. The ideal power required 10 rae the pressure is (1.5 m/s) (500 Pa) = 750W. sag an air density of 1.2 keJm?, the mass rate of flow is (5 m/s) (1.2 balm) = 18 kas, Since the area of the fan outlet is 05170 289) = 0.149 GF the velocity is (1.5 m3/5)(0.149 m2) = 10.1 mls. The power required (0 Mrovide the Kinetic energy to the airis(1-8 ks) (101°/2) «91 ‘igure 6-15 shows the power required by the fn at the operating point {2 bbe 1.2 RW; the fan efficiency is therefore 750+ 91 1200 70% ” prod ryx209905 ee FAN AND DUCT SYSTEMS 121 ‘Wheel or impeller Airoutle Figure 6-14 A centrifugal fan. periphery of the wheel and directs it toward the outlet. A fan may have a single or double inlet, depending upon whether the air enters the impeller from one or from both sides. The usual direction of discharge is horizontal, but certain applications call for discharge to be nonhorizontal. Four types of blading are common in centrifugal fans, radal, forwardcurved, backward-curved, and airfoil. The forward-curved blade fan is commonly used in low- pressure air-conditioning systems and is the only type considered in this chapter. [Aiefoil- or backward-curved-blade fans are used in high-volume cr high-pressure sys- tems where increased efficiency is important. "The typical shape of the pressure low characteristics of a forward-curved-blade fan operating at various speeds is shown in Fig. 6-15. The characieristic dip in static pressure at low flow rates results! because the channels between the blades partially fill with eddies rather than directed flow. "The power required by the fan is also shown on Fig. 6-15. Tue power ideally re- ‘quired has two components: that needed to saise the pressure and that needed to provide the kinetic energy setting the air in motion. The power required to raise the pressure derives from the expression for an isentropic compression wyv dp Where w = mass rate of flow, ke/s = specific volume, m3/kg ‘The specific volume changes little because the changes in absolute pressure are small in fan, so v can be removed from within the integral, giving the ideal power required to ralse the pressure as Power to ruse presure = Ql, ~P,)W 627) where = volume rate of flow, m/s py Py = pressure rise, Pa ‘Since the power required to provide kinetic energy to the air is wV2/2, the combin:

You might also like